Anda di halaman 1dari 55

PROFESSIONAL RESPONSIBILITY OUTLINE

I. The Regulation of Lawyers


A. Admission to Practice
1. Requirements for Admission
a. Graduation from an accredited undergraduate college
b. Graduation from a law school that meets the states educational standards
(usually meaning accreditation by the ABA)
c. Submission of an application for admission to the state bar
d. A passing score on the state bar examination (including the MPRE)
e. Finding of good moral character and fitness to practice law
(i) No real definition.
(ii) Purpose: to assess whether the applicant will practice law in an honest
and competent manner
(a) Justification: Lawyering is a privilege, not a right.
(iii) General Criteria
(a) Includes residence and employment history, criminal records
(arrests, convictions, citations), traffic records, credit history,
records of any litigation in which the applicant has been a party.
(b) Recent Court Decisions
(i) Admission Denied for...
(a) international drug smuggling (MA)
(b) sexual relations with minors (OK)
(c) declaration of bankruptcy (MN)
(d) copying another students exam answers (SD)
(e) plagiarism (WI)
(f) pattern of hostile and disruptive conduct (NB)
(ii) Admission Granted for...
(a) declaration of bankruptcy (FL)
(b) shoplifting and misrepresentation of debt (OR)
(iv)The Character Questionnaire
(a) Be scrupulously honest in everything you say.
(i) Information you disclose must be consistent with information
disclosed on law school application.
(b) Options
(i) Admit and explain
(ii) Deny
(iii) Refuse to answer and state grounds
(a) Fifth Amendment?
(b) Privacy?
(iv)Call an expert
(v) File a lawsuit
(v) Mental Health of Applicants
(a) Includes whether an applicant has received outpatient treatment for
any mental or emotional condition; whether an applicant has ever

1
been hospitalized for psychiatric reasons; abuse of drugs or alcohol
and/or treatment for substance abuse.
(b) Rose Gower Story (applied in CT, application held up because of
depression treatment after high school and therapy since)
(vi)Misconduct During Law School
(a) John Mustafa Story (sketchy financial dealings in Moot Court bank
account at UCLA Law denied license to practice law!)
(vii) Law School Discipline: A Preliminary Screening Process
B. Professional Discipline
1. The History and Process of Lawyer Discipline
a. In most states, the highest court runs the disciplinary system.
b. Process:
(i) Complaint filed by client or by a lawyer
(ii) Bar counsel investigates complaint
(a) File closed if complaint does not warrant charge, OR
(iii) Charges filed if warranted by investigation
(iv)Hearing committee conducts hearing, makes factual findings,
recommends sanction
(v) Hearing committee decision reviewed by judicial agency and/or by
highest state court. Reviewing body makes final decision on sanction.
2. Grounds for Discipline
a. Violation of the applicable ethics code
(i) whether or not the violation occurs in the course of law practice (e.g.,
domestic violence, failure to pay child support, drunk driving)
(ii) whether or not the violation occurs in the state in which the lawyer is
admitted
(a) A lawyer who is admitted in more than one state must report to the
other states where she is admitted if discipline is imposed in one of
the states.
b. Any criminal act that violates an ethical rule or that reflects dishonesty,
untrustworthiness, or lack of fitness to practice law.
(i) even if no criminal charge is filed or the lawyer is acquitted of the
charge in a criminal proceeding
c. Especially applicable to Law Professors
(i) Plagiarism
(ii) Neglect of teaching responsibilities
(iii) Manipulation of grades
(iv)Aggressive or discriminatory behavior
(v) Dishonest behavior
C. Reporting Misconduct by Other Lawyers
1. The Duty to Report Misconduct
a. General Rule Under 8.3(a): If a lawyer knows that another lawyer has
violated the Rules in a way that raises a substantial question as to that
lawyers honesty, trustworthiness or fitness to practice law, the lawyer
must inform the appropriate professional authority.

2
(i) Knowledge Standard: Objective; whether a reasonable lawyer in the
circumstances would have a firm opinion that the conduct in question
more likely than not occurred (more than mere suspicion!).
(ii) Himmel Case: One of the few cases where an attorney was disciplined
under 8.3(a), but it is a unique case decided under the antiquated
Model Code.
b. ONLY Two Exceptions, Under 8.3(c)
(i) A report need not be made if it would reveal information required to be
kept in confidence under Rule 1.6. A lawyer should encourage a client
to waive confidentiality and permit reporting if that would not
substantially prejudice the client. (See Comment 2.)
(ii) A lawyer assisting a lawyer who is in a treatment program is not
required to report. (See Comment 5.)
c. General Notes
(i) If a lawyer learns of misconduct during an adversary proceeding, the
lawyer may defer reporting the misconduct until the proceeding has
concluded if deferral is necessary to protect a clients interests
(Restatement 5, comment i).
(ii) A lawyer who fails to report serious misconduct by another lawyer
may be subject to discipline.
2. Lawyers Responsibility for Ethical Misconduct by Colleagues and Superiors
a. Rule 5.1(a)
(i) Requires managing partners to set up systems to check for conflicts of
interest and to provide continuing education in legal ethics.
(ii) Applies to legal services organizations, corporate legal departments,
and government agencies (per Rule 1.0(c)s definition of law firm).
b. Rule 5.1(b)
(i) If a subordinate lawyer commits an ethical violation, the supervising
lawyer is not responsible for that violation if the supervisor did not
direct or know about it. But the violation could reveal a breach of the
supervisors duty under Rule 5.1(b) to make reasonable efforts to
prevent violations. (Comments 6 and 7.)
c. Rule 5.1(c)
(i) Partners have at least indirect responsibility for all work being done
by the firm as well as supervisory responsibility for work being done
by subordinate lawyers on matters for which they are responsible.
(Comment 5.)
d. Rule 5.1(c)(2)
(i) The directly supervising partner is not the only partner who may be
responsible for a violation. Any partner in the firm who knows of the
improper conduct and fails to take action to reduce or prevent the harm
also commits a violation.
3. Legal Protections for Subordinate Lawyers
a. Rule 5.2(a)

3
(i) If a supervisor directed the action, the subordinate lawyer may be able
to prove that she did not actually know that the action was improper.
(Comment 1.)
b. Rule 5.2(b)
(i) If the supervisor reasonably thinks the conduct is proper, the
subordinate may undertake the action even if she believes otherwise.
If the supervisor turns out to be wrong, however, the supervisor could
be disciplined. (Comment 2.) If the supervisor was SO wrong that the
subordinate lawyers belief that the action was proper was not
reasonable, the subordinate may also be disciplined.
c. General Notes
(i) The law does not allow discipline of all of the lawyers in a firm, or
even all of the partners, if one of the firms associates violates a
disciplinary rule.
(a) Contrast law firm rule (NY and NJ).
d. Options for Associates Told to Do Something Unethical
(i) Accept the directions of the superior.
(ii) Argue with the superior.
(iii) Discuss the problem with another superior.
(iv)Do more research or investigation to try to clarify the problem.
(v) Ask to be relieved from work on the matter.
(vi)Resign (or be fired) from employment.
e. Lawyers fired for insisting on compliance with ethical rules are allowed to
sue from wrongful discharge in some states but not others.

II. Client Confidentiality


A. The Basic Principle of Confidentiality
1. General Rule of 1.6(a): Protection of Information Relating to the
Representation of a Client
a. Policy: to facilitate open communication between lawyers and clients
(i) If lawyers were not bound to protect clients secrets, clients might be
more reluctant to share their secrets with their lawyers.
(ii) Concerned with client anonymity
b. Comment 4 to Rule 1.6
(i) The prohibition against revelation of confidential information also
applies to disclosures by a lawyer that do not in themselves reveal
protected information but could reasonably lead to the discovery of
such information by a third person.
(ii) A lawyers use of a hypothetical to discuss issues relating to the
representation is permissible so long as there is no reasonable
likelihood that the listener will be able to ascertain the identity of the
client or the situation involved.
c. What is Confidential?
(i) All information relating to the matter on which the lawyer is
representing the client is confidential, except information that is

4
generally known (which is distinct from public knowledge, as in
available through the courts).
(a) Second Attorneys: If your client fires you and hires new counsel,
and the new counsel asks for all the information you have for that
client, you are authorized only to provide copies of pleadings!
Thats it!
(ii) Personal information relating to the client that the client would not
want disclosed
(a) Sometimes Confidential: the fact that a lawyer is representing the
client (i.e., that the person is a client at all his identity)
(iii) Information learned from the client, and information learned from
interviews, documents, photographs, observation, or other sources
(iv)Information acquired before the representation begins (such as during
a preliminary consultation) and after the representation terminates
(v) Notes or memoranda that the lawyer creates relating to the matter
(vi)Exception: You can talk generally about your caseload, but dont use
client names.
d. Compare to Restatement
(i) Restatements Lower Standard: prohibits revelation of such
information only if there is a reasonable prospect that doing so will
adversely affect a material interest of the client or if the client has
instructed the lawyer not to use or disclose such information.
(a) Adverse Effects: The Restatement defines adverse effects to
include
(i) frustration of the clients objectives in the representation;
(ii) material misfortune, disadvantage, or other prejudice to the
client;
(iii) financial or physical harm to the client; or
(iv)personal embarrassment to the client.
(ii) Closer to the norm of the profession
e. Consequences of Failing to Protect Confidences
(i) Subject to professional discipline
(ii) Liable in tort or contract for negligent or intentional breach of
fiduciary duty
(iii) Disqualified from representation of one or more clients
(iv)Enjoined by a court from further revelation
B. Exceptions Provided by Rule 1.6(b)
1. Analysis: To be excepted from the General Rule to protect confidences, the
lawyer can
a. Obtain the clients informed consent (under 1.6(a)), OR
b. Have implied authorization (in order to carry out the representation)
(under 1.6(a)), OR
c. Fall within one of the following 6 permissive exceptions (only to the
extent that the lawyer believes it is reasonably necessary) (under 1.6(b))
(i) But before doing this, the lawyer should try to persuade the client to
take action that will obviate the need for disclosure. (Comment 14)

5
(ii) NB: The disclosure should be as narrow in content as possible and
should be made to as few other people as possible! (Comment 14)
2. Exception 1: To Prevent Reasonably Certain Death or Substantial Bodily
Harm
a. Reasonably Certain: if the harm will be suffered imminently or if there
is a present and substantial threat that a person will suffer such harm at a
later date if the lawyer fails to take action necessary to eliminate the threat
(Comment 6)
b. It doesnt matter whether the possible harm will be perpetrated by the
client or by another person.
c. Policy: recognizes the overriding value of life and physical integrity.
d. Past Criminal Acts: The rules require lawyers to keep information about
past crimes by clients confidential.
3. Exceptions 2 and 3: Fraud Causing Substantial Financial Injury
a. Fraud: deliberate deception
(i) If an act is fraudulent under any substantive or procedural legal
standard relevant to the act in question in the applicable jurisdiction, it
is reasonably to assume that it is fraudulent under the rules.
(ii) Fraud is defined slightly differently under
(a) Tort Law (intentional misrepresentation of fact or law intended to
induce reliance that would result in financial loss)
(b) Criminal Law (intentional misrepresentation of material facts, no
harm necessary)
(c) Contract Law (intentional misrepresentation, includes material
omissions)
(iii) Omissions and half-truths as well as false representations can
constitute fraud.
(iv)Under the Model Rules, a lawyers omission may be fraudulent if the
lawyer intended to deceive another person.
b. 3 Policies Balanced
(i) To encourage frank communication between clients and lawyers
(ii) To prevent harm to the public
(iii) To protect the integrity of the profession by allowing lawyers to
blow the whistle if their own work is being used to commit crimes or
frauds
c. Sarbanes-Oxley Act: requires lawyers who practice before the SEC or who
advise companies regulated by the SEC to report any information about
securities fraud to the highest officials of the corporation.
(i) Exceptions 2 and 3 apply to all lawyers, however (its broader).
d. Exception 2: To Prevent the Clients Future Commission of a Crime or
Fraud Reasonably Certain to Result in Substantial Financial Injury, In
Connection With Which the Client Would Use the Lawyers Services
e. Exception 3: To Prevent, Mitigate, or Rectify Substantial Financial Injury
Post-Commission of Clients Crime or Fraud, In Connection With Which
the Client Used the Lawyers Services

6
(i) This exception does not apply when a person who has committed a
crime or fraud thereafter employs a lawyer for representation
concerning that offense. (Comment 8)
f. Two Times When Mandatory to Reveal Confidences
(i) See also Rule 4.1(b): In the course of representing a client, a lawyer
shall not knowingly...fail to disclose a material fact when disclosure is
necessary to avoid assisting a criminal or fraudulent act, unless
disclosure is prohibited by Rule 1.6.
(a) NB: Rule 1.6 now permits revelation of confidential information to
prevent, mitigate, or remedy some criminal and fraudulent client
conduct. This means that in any situation in which a lawyers
failure to reveal would constitute assisting a criminal or
fraudulent act, Rule 4.1 requires a lawyer to reveal the
information (mandatory disclosure).
(ii) See also Rule 3.3: if a lawyer had offered false testimony, failure to
disclose that fact to the court would constitute assisting fraud.
g. See also Rule 1.16(a): requires a lawyer to withdraw from representing a
client if continued representation would result in a violation of the rules.
(i) Noisy Withdrawal: while withdrawing from representation, a lawyers
disaffirmation of a document prepared by the lawyer, indicating he no
longer vouches for the documents accuracy, thus putting another party
on notice that the lawyers client may be trying to commit fraud.
(ii) Example: Where lawyer prepares loan form for client on basis of false
information, and learns that client plans to resubmit the document for
financial gain.
4. Exception 4: To Obtain Advice About Legal Ethics
a. Policy: Compliance with the Rules is more important than protecting
client confidences!
b. The lawyer is not required to use a hypothetical to keep the clients
identity a secret. He may share confidences and documents, if necessary.
The adviser must commit to maintain the confidences, presumably.
5. Exception 5: To Protect the Lawyers Interests (Self-Defense Exception)
a. When A Lawyer May Use This Exception
(i) To establish a claim against a client for unpaid fees
(ii) To defend against a claim of malpractice or other claim of civil
liability against the lawyer
(iii) To defend against a disciplinary proceeding
(iv)To defend against a criminal charge
b. NB: The lawyer may be allowed to reveal confidences even if he is not
alleged to be the primary wrongdoer and even if the allegation is not made
by h is client (e.g., if a third person is alleging fraud by both client and
lawyer).
c. Timing: The lawyer need not wait for a complaint to be filed against him
before the right to self-defense under this exception kicks in. (Comment
10)

7
d. Procedure: If a lawyer needs to reveal confidences to protect her own
interests, she must take steps to avoid the need for revelation, to limit its
scope, or to limit the dissemination of the information.
e. Client Notification: The lawyer should notify the client before using
confidential information in self-defense and should seek solutions that do
not require the lawyer to make the revelation, but the lawyer may use the
information even if the client does not consent.
6. Exception 6: To Comply with Other Law or a Court Order
a. If a court order or other law requires a lawyer to reveal confidences, then
if that other law supersedes the ethics rules, the lawyer may reveal the
information. (Comments 12 and 13.)
(i) Example: Most states require anyone who knows of ongoing child
abuse to report it to welfare agencies.
b. The lawyer should try to resist the revelation as required by some other
law, if possible. (Comment 12)
C. Use or Disclosure of Confidential Information for Personal Gain or to Benefit
Another Client
1. Rule 1.8(b): A lawyer shall not use information relating to representation of a
client to the disadvantage of the client unless the client gives informed
consent, except as permitted or required by these Rules.
a. Comment 5: Rule 1.8(b) doesnt prohibit use of information learned in
course of client representation to the advantage of other clients, so long as
the first client is not disadvantaged by the use of the information or gives
informed consent.
D. Talking to Clients About Confidentiality: A Policy Dilemma
1. Most clients would prefer to know at the beginning of the relationship that a
lawyer might need to reveal confidential information in a particular matter,
before they reveal a great deal of sensitive, confidential information.
2. On the other hand, a well-counseled client might be less open with his or her
lawyer.
III.The Attorney-Client Privilege and the Work Product Doctrine
A. Confidentiality and Attorney-Client Privilege, Compared
1. Ethics Law Versus Evidence Law
a. Confidentiality: The duty to protect confidences is imposed by the ethical
rules, violation of which can result in discipline.
(i) Confidentiality requires lawyers to protect confidential information
whether or not someone is trying to compel the disclosure of
information.
b. Privilege: evidence law which governs what kinds of evidence can be
admitted in court.
(i) Privilege rules provide that neither lawyer nor client may be compelled
to testify in court about protected communications.
(ii) If a lawyer has relevant information and no privilege applies, he can be
required to testify.
2. Similarity in Policy: A legal system in which advocates speak for clients will
work best if clients feel free to speak openly to their attorneys.

8
3. Difference in Scope
a. Confidentiality: covers all information relating to the representation that
a lawyer obtains (Rule 1.6(a)
b. Privilege: covers only a subset of that information, that part that involves
communications between lawyer and client in which the client is seeking
legal advice or other legal services
4. Different Methods of Enforcement
a. If a lawyer violates a confidentiality rule, the lawyer might be subject to
discipline.
b. If a lawyer seeks information that another lawyer claims is privileged, the
second lawyer might move to quash a subpoena or object to compliance
with a discovery request, claiming privilege.
5. Source of the Privilege: State Common Law
a. The exact scope of the privilege is different in each state.
b. When federal courts apply state law, they also apply state privilege rules.
B. The Four Elements of Attorney-Client Privilege
1. Communication
a. Examples: telephone call, memorandum, letter, fax, e-mail, IM exchange,
or any other mode of exchanging information.
(i) Non-Examples: Client identity and fee arrangements
b. NB: The privilege protects only against disclosure of the
communication itself, not against disclosure of the underlying facts
that might have been communicated.
c. Communications with a prospective client or lawyer are privileged.
(i) Policy: If there were no such privilege, it would be very hard for a
client to tell a lawyer the information, in confidence, to enable the
lawyer and client to decide whether the lawyer would represent the
client.
2. Privileged Persons
a. Communications with agents of an attorney (secretaries, paralegals,
investigators, etc.) are also privileged.
b. Communications with agents of the client (e.g., language interpreter, third
person needed for psychological support, parents of a minor child-client,
etc.), assuming the third persons presence is not considered a waiver of
the privilege, are also privileged.
c. A lawyers or secretarys notes of a privileged conversation are privileged,
just as if the client had made her own notes and brought them to the
lawyer in the form of a memorandum.
3. Communication in Confidence
a. The client must reasonably believe that the communication is confidential.
(i) There is no privilege for communication that occurs in the presence of
other people!
b. The circumstances of the communication must be reasonably private.
c. The disclosure to a third person of the contents of privileged
communications (by either the attorney or the client) technically waives

9
the privilege (but this is never of real practical importance, as the opposing
counsel rarely knows about these disclosures!).
4. Communication for the Purpose of Seeking Legal Assistance
a. No exchange of promises or money is necessary to create a lawyer-client
relationship; so long as the client is seeking legal advice, that
communication is privileged.
b. If a conversation is partly legal advice and partly personal/business advice,
only the part that is legal advice is privileged.
c. The privilege protects communications from the client to the lawyer and
from the lawyer to the client.
d. The privilege also protects a confidential memo a lawyer writes for his
files or for a co-counsel that includes a record of a privileged
communication with the client.
(i) If a communication is a document, it must have come into existence at
the same time as or later in time than the clients first communication
with the lawyer.
C. Consequences If a Communication is Privileged
1. The lawyer cannot be forced to testify.
a. The lawyer may not testify over the clients objection; the privilege
belongs to the client!
2. The client cannot be forced to testify.
a. However, the client may waive the privilege.
3. Paper and electronic records of a privileged communication are also protected.
a. Only the communication, not the underlying information, is protected.
D. The Crime-Fraud Exception
1. No privilege if a client (individual or corporation) goes to a lawyer and
discusses intended fraudulent or criminal activity (as opposed to seeking legal
advice)!
a. The clients intention to perform a criminal or fraudulent act triggers the
crime-fraud exception.
(i) It does not matter whether the client knows that the act is wrongful.
(ii) It also does not matter whether the lawyer knew that the act the client
intended to commit was wrongful (because not given enough facts by
the client).
(iii) If, however, the lawyer has criminal intent, but the client does not,
the conversation remains privileged! Only the clients intentions are
relevant to the privilege!
b. Distinction: If a client instead asks for advice regarding whether a certain
act is permitted under law, the communication is privileged (at least in
most states).
2. No privilege for a conversation if the client later uses the lawyers advice or
services to commit a crime or a fraud.
a. Distinction: If a client asks his lawyer for advice about a past act that was
criminal or fraudulent, such communication is privileged so long as the
past act is really past; the crime-fraud exception does not apply to past
crimes or frauds.

10
(i) If there is a continuing crime or fraud that results from a past act, there
is no privilege.
3. Procedure for Challenging the Privilege on the Basis of the Exception
a. The opposing lawyer, who believes the crime-fraud exception applies,
must request documents based on guesses about what might exist or what
the documents might contain.
E. The Death of the Client
1. Swidler & Berlin v. United States: held that attorney-client privilege continues
even after the death of the client (reasoning that posthumous disclosure of
such communications may be as feared as disclosure during the clients
lifetime for purposes of maintaining reputation or possible harm to friends or
family).
F. Waiver
1. Express Waiver by Client
a. Privilege may also be waived by the clients lawyer if the waiver has been
authorized by the client.
2. Waiver by Inaction
a. Example: Client is being deposed (or cross-examined) by opposing
counsel, who asks for privileged information. If clients lawyer does not
object on grounds of the privilege, and the client answers the question,
then the privilege is held to be waived.
3. Waiver by Revealing Privileged Communication to a Nonprivileged Person
a. Nonprivileged Persons: In some states, clergy, physicians, spouses, and
psychologists do not constitute nonprivileged persons.
b. Joint Representation: If two clients hire a lawyer jointly, they are
considered common clients with a common privilege.
(i) If these two clients hire the lawyer separately, and the lawyer wants
the privilege to apply to conversations where both are present, the
lawyer should obtain their agreement to participate in a common
representation in which the confidential information is shared among
the three of them.
c. Fax Problem: If the attorney did not take adequate precautions to protect
against inadvertent disclosure via fax, then the privilege is waived. (Rule
established by Judge Young in MA.)
4. Waiver by Putting Privileged Communication Into Issue
a. Example: If the client sues the lawyer for malpractice and asserts that the
lawyer gave her certain incorrect advice, the lawyer may reveal the details
of the relevant conversations for the purpose of self-defense (under Rule
1.6(b)(5)), and the privilege is waived. (Restatement 83(2).)
5. Waiver as to Conversation by Disclosure of Part of It
a. Partial disclosure may be found to be a waiver of the privilege as to that
part of the conversation that relates to the subject matter on which the
client volunteered testimony.
b. Policy (Subject Matter Test): to prevent the client from offering
misleading testimony by revealing only a half-truth.
6. Compliance With Court Orders

11
a. Turning the information over in response to a court order does not waive
the issue for purposes of appeal or other litigation.
G. The Work Product Doctrine
1. Work Product Prepared in Anticipation of Litigation
a. Applies to documents that a lawyer prepares or collects while working on
pending litigation or on a mater in which the lawyer knows that a lawsuit
is about to be filed.
b. Protects some types of documents that are not covered by the attorney-
client privilege because they do not relate to communications between
lawyer and client.
c. Not absolute (more likely enforced if document contains responses to
lawyers questioning of witness, for example, than if document is written
entirely by witness and mailed to lawyer).
2. Origins of the Work Product Rule
a. First recognized in Hickman v. Taylor (1947)
3. Materials Not Created or Collected in Anticipation of Litigation
a. If certain information would have been collected routinely but was also
collected because litigation was anticipated, most courts will deny
protection to the information.
b. If a client gives a lawyer a set of documents that are relevant to impending
litigation, the documents are not usually protected by the work product
doctrine.
4. A Qualified Protection
a. A judge can order disclosure of written or oral information otherwise
protected by this doctrine if the opposing party can show substantial
need for the material and that the opposing party is unable without
undue hardship to obtain the substantial equivalent of the material by
other means.
5. Protection of the Lawyers Mental Impressions
a. The doctrine gives stronger protection to work product that reveals the
lawyers thoughts, strategies, or mental impressions than it does to
ordinary work product.
b. A lawyers own notes of his own opinions, theories, observations, or
feelings are immune from discovery.
6. Protection of Work Product, Not Underlying Information
a. A lawyer can often get the information contained in protected documents
from the original witnesses or sources.
b. The doctrine prevents freeloading on an opponents work, but it does not
enable the opponent to close off a lawyers sources of information by
getting there first!
IV. Relationships Between Lawyers and Clients
A. Lawyer-Client Relationship
1. Creation
a. Rendering of Legal Advice: If a person seeks legal advice or legal services
from a lawyer, and the lawyer gives legal advice or provides legal
services, the person may thereby become a client.

12
(i) Ok to give legal information
(ii) Relationship is created to the extent that a client would reasonably rely
on the lawyers advice to his detriment.
b. Not Required
(i) Payment of fee
(ii) Signing a written agreement
2. Choosing Clients
a. General Rule: Lawyers are allowed to be picky; they dont have to accept
any particular clients.
(i) Caveat #1: Rule 6.1 encourages lawyers to provide pro bono legal
services.
(ii) Caveat #2: Rule 6.2 directs lawyers to accept court-assigned
representation of an indigent criminal defendant, even if the court does
not have the resources to pay the lawyer for the work, except for good
cause.
(iii) Caveat #3: A lawyer may not discriminate on the basis of race,
religion, nationality, sex, age, disability, or another protected category
in her decisions about which clients to represent.
B. Lawyers Responsibilities as Agents
1. General Rule: Because lawyers are agents of their clients, clients are generally
bound by what the lawyer does or fails to do, regardless of the clients own
actions or culpability.
a. Examples: waiver of defense; waiver of privilege; agreement to buy or sell
property
2. Actual vs. Apparent Authority
a. Two Types of Actual Authority
(i) Express Authority: where the client gives the lawyer explicit authority
to act on the clients behalf
(ii) Implied Authority: where the client gives the lawyer a general
instruction that implicitly allows the lawyer to take certain actions on
the clients behalf
(a) NB: Just by asking a lawyer to represent him, a client impliedly
authorizes the lawyer to take action that is reasonable and
calculated to advance the clients interest
(b) Settlements: require actual or apparent authority!
b. Apparent Authority
(i) Restatements Three Elements
(a) Client has actually retained a lawyer.
(b) Clients acts caused a reasonable observer to infer authority.
(i) Objective standard!
(c) Third party in fact believed that there was authority.
(ii) Examples
(a) when a client tells a third party that the clients lawyer has the
authority to settle a claim on his behalf (prompting the third party
to rely on the lawyers subsequent actions)

13
(b) when a principal places an agent in a position that causes a third
person reasonably to believe that the principal had given the agent
express authority
(c) The more conduct exhibited by the client, the more likely apparent
authority will be found.
(iii) Nonexample
(a) when a lawyer makes a statement to a third party that she is
authorized to act
C. Lawyers Duties of Competence, Honesty, Communication and Diligence
1. Competence
a. MacCrate Report
(i) Ten Lawyering Skills (with which a well-trained generalist should be
familiar)
(a) problem-solving
(b) legal analysis and reasoning
(c) legal research
(d) factual investigation
(e) oral and written communication
(f) counseling
(g) negotiation
(h) litigation and alternative dispute resolution procedures
(i) organization and management of legal work
(j) recognizing and resolving ethical dilemmas
b. Lack of Experience: If the lawyer compensates for inexperience through
study or affiliation with another lawyer, he may accept legal work that
requires knowledge of an area of law in which he has no experience.
(i) Comment 2 to Rule 1.1 explains that a lawyer can take on a matter in
an unfamiliar field if the lawyer has the (1) time and (2) resources to
get up to speed.
c. Competence in Criminal Trials
(i) Sixth Amendment: requires that a criminal defendant be provided with
a lawyer whose work meets at least the minimum standard of being
effective.
(ii) Many judges are very reluctant to overturn a criminal conviction
because a different lawyer might have done a better job for a
defendant.
(a) Strickland v. Washington (1984)
(i) Two Elements of Ineffective Representation
(a) #1: deficient performance by counsel
(i) a showing that counsel made errors so serious that
counsel was not functioning the counsel guaranteed
by the Sixth Amendment
(b) #2: which prejudiced the defense
(i) a showing that counsels error were so serious as to
deprive the defendant of a fair trial, a trial whose result
is reliable

14
(ii) assistance was so poor that there is a reasonable
probability that better representation would have made
a difference
2. Candor and Communication
a. Relevant Questions in Assessing the Justifiability of a Lie
(i) Is the subject lied about either trivial or private?
(ii) Is anyone harmed by the lie?
(iii) Is the purpose of the lie to protect someone?
(iv)Does the person lied to have a right to know (or a strong interest in
knowing the truth)?
(v) If there is a reason to tell a lie, can the problem be solved without
lying?
(vi)If you tell this lie, will you need to tell other lies to cover up the first
one?
b. Truth Versus Truthfulness
(i) It is not dishonest to make a false statement if you sincerely believe
that the statement is true.
c. Honesty and Communication Under the Ethics Rules
(i) The ethics rules explicitly direct lawyers not to lie to tribunals or to
persons other than clients. (Rules 3.3(a) and 4.1)
(ii) The drafters intended that lawyers should generally be truthful, and
this intention extends to clients as well as to others.
(iii) Rule 8.4(c): prohibits a lawyer from engaging in conduct
involving dishonesty, fraud, deceit, or misrepresentation.
(iv)Rule 1.4: requires that a lawyer provide information to a client about
matters that require informed consent, about which a client must make
a decision, about the status of a matter, and about matters on which the
client has requested information.
(a) (a)(2): Lawyers have discretion regarding the means to carry out
the representation, but the objectives are to be set by the client.
(b) (a)(3): Status includes significant developments affecting the
timing or substance of the representation (Comment 3).
(c) (a)(4): If a lawyer cannot respond promptly, he should explain
when a response may be expected, and client telephone calls
should be promptly returned or acknowledged (Comment 4).
(d) (b): The lawyer should give the client enough information to
participate intelligently in decisions about objectives and means,
but a lawyer ordinarily will not be expected to describe trial or
negotiation strategy in detail (Comment 5).
d. Civil Liability for Dishonesty to Clients
(i) A client injured by a dishonest lawyer may sue the lawyer in tort for
(a) Fraud
(b) Breach of Fiduciary Duty: the lawyers failure to act consistently
with the trust that a client reposes in a lawyer because the lawyer
has special skills and knowledge.
(i) usually results from disloyalty

15
(ii) An attorney who willfully and deliberately lied to, misled, and stole
from his clients could be sued for punitive as well as compensatory
damages (even though punitive damages are not available for the
ordinary negligence-based tort of malpractice).
e. Diligence
(i) One fundamental duty of lawyers toward clients is to do the work that
they have been hired to do, and to do it without undue delay.
(ii) The Model Rules replaced the requirement of zeal with one of
diligence and demoted the language on zeal to the comments.
(a) Comment 1 urges zealous advocacy, but it states that a lawyer is
not bound...to press for every advantage that might be realized.
(iii) Lawyers are not excused from the duty of diligence if they leave a
law firm, unless they formally withdraw from representation.
f. Contractual Duties
(i) If a lawyer violates duties that are imposed only by contract, the
lawyer may be subject to discipline for those contractual violations.
(See Restatement 16, comment f, relating to an earlier version of
Rule 1.3.)
D. Who Calls the Shots?
1. Rule 1.2(a): The Competent Adult Client
a. Client decides the objectives of representation.
(i) Civil Case: Client decides whether to settle
(ii) Criminal Case: Client decides whether
(a) to plead guilty
(b) to waive jury trial
(c) to testify
(iii) Lawyers Goal: get for your client what she would get if she were
free from interferences and distortions
(a) If client is stuck and cannot decide how to proceed, the lawyer may
not tell him what to do; the client must decide objectives on basis
of his own values, risk-aversion, and feelings! (No rule says this
though.)
(i) Lawyers may give their opinions, so long as they lead in with
disclaimers (e.g., my risk-aversion may be different than
yours...).
b. Lawyer must consult client as to means to pursue objectives.
(i) Disagreements: When lawyers and clients disagree about the means to
be employed, the rule does not prescribe how such disagreements are
to be resolved.
(a) Solution: Restatement 21, comment e, suggests that unless a
lawyer and client have agreed otherwise, the lawyer, not the client,
should make decisions that involve technical legal and strategic
considerations difficult for a client to assess.
(i) Jones v. Barnes (1983): held that attorneys are not required to
make legal arguments requested by their criminal-defense

16
clients (an attorneys duty is to take professional responsibility
for the conduct of the case, after consulting with his client...)
2. Rule 1.2(d): A lawyer may not assist his client in conduct he knows to be
criminal or fraudulent.
a. Comment 10: Sometimes the only way you can avoid assisting fraud once
you have learned that your work has already been used to commit fraud is
to (1) withdraw and then (2) disaffirm any opinion, document, affirmation
or the like (a noisy withdrawal).
3. Contracts to Change the Duties Owed to Clients
a. Lawyers and clients may agree that lawyers do not owe clients some of the
duties otherwise imposed by the rules.
(i) Reasoning: The Model Rules are merely default rules.
b. Impermissible Limitations
(i) A lawyer may not enter into an agreement waiving the duty of
competent representation.
(ii) A lawyer is also barred from making an agreement prospectively
limiting the lawyers liability for malpractice unless the client is
independently represented in making the agreement. (Rule 1.8(h)(1))
4. Rule 1.14: Clients With Diminished Capacity
a. (a): encourages lawyers to maintain normal lawyer-client relations with
clients who may have some degree of diminished capacity.
b. (b): acknowledges that in some situations, a lawyer needs the flexibility to
assume a more paternalistic role to protect the client from some harm.
c. Comment 6: suggests that a lawyer can and should make some assessment
of a clients mental capacity.
(i) The rule contemplates that the lawyer may consult about the matter on
which the lawyer is representing the client with individuals or
entities other than the client, such as members of the clients family
or public agencies (but it does not explain how the lawyer should
know whether to do so!).
d. Clients Who Have Mental Disabilities
(i) If it seems necessary to give someone else authority over the client, the
lawyer might petition the court to appoint a
(a) guardian ad litem (GAL): empowered to speak for the client in a
particular legal matter (even contrary to the clients wishes)
(b) conservator: is given power to manage clients financial affairs
(c) guardian: manages the clients financial affairs and may make
medical or other personal decisions for the client (termed the
ward).
(ii) Tremblays Client-Centered Approach: Article suggests that
guardianship is legitimate in extreme cases, that reliance on family
may be appropriate, that noncoercive persuasion is justified in less
extreme cases, and that unilateral usurpation of client autonomy is
never appropriate except in emergencies.
e. Juveniles

17
(i) Rule 1.14: applies the same standards to minors that it applies to adults
with mental impairments.
(ii) Comment 1: notes that children as young as five or six years of age,
and certainly those of ten or twelve, are regarded as having opinions
that are entitled to weight in legal proceedings concerning their
custody.
(iii) From the ABAs Standards of Practice for Lawyers Representing
a Child in Abuse and Neglect Cases
(a) The lawyer has a duty to explain to the child in a developmentally
appropriate way such information as will assist the child in having
maximum input in determination of the particular position at issue.
(b) Unless there is a real risk of harm, the lawyer should advocate the
childs expressed position.
(c) If the childs attorney determines that the childs expressed
preference would be seriously injurious to the child, the lawyer
may request appointment of a separate guardian ad litem and
continue to represent the childs expressed preference, unless the
childs position is prohibited by law or without any factual
foundation.
E. Terminating a Lawyer-Client Relationship
1. Duties to Client at the Conclusion of the Relationship
a. When Concluded: Most relationships end when all the work on the
relevant matter has been completed.
(i) Continued Communication: may indicate a continued relationship; but
periodic mailings of a general newsletter would not suffice.
b. Return of Documents: At that time, the lawyer must return to the client
any papers and property to which the client is entitled and must return
any unearned payment that the client may have made. (Rule 1.16(d).)
(i) The ethics rules do not specify what types of papers must be returned
to a client or whether they must be delivered to the client even if the
client does not request them.
(ii) Restatement 46, comment c, states that a lawyer may refuse to
disclose to the client certain law-firm documents reasonably intended
only for internal review.
(iii) Exception for Unpaid Fees: If the client has not yet fully paid the
lawyers fee, or the fee is disputed, the lawyer may retain the
documents that the lawyer created for the client for which
compensation has not been received, unless retention would
unreasonably harm the client. (Restatement 43, comment c.)
2. Grounds for Termination Before the Work is Completed (Firing or
Withdrawal)
a. When the Client Fires the Lawyer
(i) A lawyer must withdraw if fired.
(ii) A lawyer must also withdraw if the lawyers illness or loss of capacity
would materially impair the representation. (Rule 1.16(a)(2).)
b. When Continued Representation Would Involve Unethical Conduct

18
(i) A lawyer must also withdraw if representation will require the lawyer
to violate the law, including the states rules of professional conduct.
(ii) If the client has already used the lawyers services to commit a crime
or fraud but continued representation will not result in new or
continuing crime or fraud, the lawyer may withdraw but is not required
to do so.
(iii) Similarly, if the client persists in a course of action that the lawyer
reasonably believes is a crime or fraud, and the lawyers services are
being used to assist this action, the lawyer may withdraw, even if the
actions have not yet been adjudicated to be criminal or fraudulent.
(iv)Finally, if the client insists on action the lawyer finds repugnant, the
lawyer may withdraw.
c. When the Lawyer Wants to Terminate the Relationship
(i) The lawyer may withdraw if it is possible to do so without material
adverse effect on the interests of the client. (Rule 1.16(b)(1).)
(a) That is, when time is not of the essence and when the matter is not
too complex, such that the client could secure new legal counsel.
d. Matters in Litigation
(i) If a lawyer has filed suit on behalf of a client or entered an appearance
in a matter in litigation, the lawyer generally cannot withdraw from
representation of the client without permission from the court that is to
hear the case.
e. When the Client Stops Paying the Fee
(i) A lawyer may withdraw if the client doesnt pay the lawyers fee, but
the lawyer must first warn the client that nonpayment will lead to
withdrawal. (Rule 1.16(b)(5).)
f. When the Case Imposes an Unreasonable Financial Burden on the Lawyer
(i) If a case turns out to impose an unreasonable financial burden on the
lawyer, the lawyer may withdraw.
g. When the Client Will Not Cooperate
(i) A lawyer may withdraw if the client makes continued representation
by the lawyer unreasonably difficult.
h. Other Good Cause
(i) Rule 1.16 permits withdrawal for other good cause.
V. Concurrent Conflicts of Interes t
A. Conflicts of Interest, Generally
1. General Rule: When a lawyer undertakes representation of a client, the lawyer
owes that client a duty of loyalty and a duty to protect confidential
information.
a. Concerns Raised By Conflicts
(i) exercise of independent professional judgment (IPJ)
(ii) potential breach of confidentiality (CONFIDENCES)
(iii) clients sense of lawyers loyalty (LOYALTY)
b. Policy: It is disloyal for the lawyer to agree to represent another client
whose interests are adverse to the first.

19
c. Policy: In the course of representing the second client, the lawyer might
(intentionally or accidentally) use or reveal confidential information
learned from the first client.
2. Possible Remedies for Conflicts
a. Withdrawing or declining to represent a new client
b. Obtaining clients informed consent (to represent both)
c. Signing agreements that limit scope of representation
d. Screening
3. Categories of Conflicts
a. Concurrent Conflict: a conflict between two present obligations of a
lawyer or between one present client and one prospective client
(i) See Rules 1.7 and 1.8.
b. Successive Conflict: a conflict between an obligation to a present client
and an obligation to a former client
(i) See Rule 1.9.
c. Imputed Conflict: a conflict between an obligation of one lawyer to a
client and an obligation of an affiliated lawyer
(i) Concurrent: conflict between a client of Lawyer A and a client of one
of As partners
(ii) Successive: conflict between a client of A and a past client of As
partner (where the former client may have been represented by the
same firm or a former firm of As partner)
(iii) See Rule 1.10 (and Rule 1.7, for waiver of conflict by client)
d. Conflicts for Government Lawyers: who move between public and private
sectors
(i) Successive: where a lawyers new clients interests conflict with the
interests of his former employer
(ii) Imputed: where work done by other lawyers in a firm conflicts with t
he interests of a lawyers former employer
(iii) See Rule 1.11
B. General Principles in Evaluating Concurrent Conflicts
1. Application: Rule 1.7 provides protections for all current clients!
2. General Rule (Rule 1.7(a)): One cannot represent a client if it would cause a
concurrent conflict of interest!
a. Two Disjunctive Elements of a Concurrent Conflict (only 1 necessary)
(i) Direct Adversity (Rule 1.7(a)(1))
(a) Definition: if the lawyers conduct on behalf of one client requires
the lawyer to act against the interests of another current client
(b) Examples
(i) A lawyer representing client A in one matter sues client A in a
second matter on behalf of client B.
(a) Suing a Current Client: You cannot sue a current client
without consent of both clients!
(ii) The lawyer acts adversely to a client in litigation (e.g., by
cross-examining him).

20
(iii) Outside of litigation, a lawyer is asked to undertake
adversarial negotiation against another client. (Comments 6
and 7)
(ii) Significant Risk of Material Limitation in Representation (Rule 1.7(a)
(2))
(a) that is, a significant likelihood that a difference in interests will
eventuate and, if it does, [that] it will materially interfere with the
lawyers independent professional judgment (Comment 8)
(i) Two-Prong Inquiry
(a) How likely is it that a difference in interests will
eventuate?
(b) If there likely is such a divergence, would it materially
interfere with the lawyers advice to or representation of a
client?
(ii) NB: A mere possibility of harm is insufficient to present a
conflict.
(b) Sources of Material Limitation
(i) obligations to a present client
(ii) obligations to a former client
(iii) obligations to someone else to whom the lawyer owes a
duty (e.g., as a trustee, an executor, or a member of a corporate
board of directors)
(iv)obligations to someone other than the client who is paying the
lawyers fee
(v) obligations to the lawyers own financial, employment,
personal or other interests.
3. Exceptions to the General Rule (Rule 1.7(b)): If a concurrent conflict exists, a
lawyer can still represent a client if...
a. Four Conjunctive Requirements (all 4 necessary to gain exception)
(i) (b)(1): he reasonably believes he can still provide competent and
diligent representation
(a) Objective Standard: The question is what a reasonable lawyer
would think.
(b) Considerations
(i) Are the matters involved factually interrelated?
(ii) Do the two parties have very divergent interests (or could
they)?
(iii) Does the lawyer have a friendship or bond of professional
loyalty toward one of the clients?
(ii) (b)(2): the representation is not prohibited by law
(a) Example: Some state laws prohibit a lawyer from representing
more than one client in a capital case. (Comment 16, Comment
23)
(iii) (b)(3): the adverse parties are not litigating against each other
(a) Example: two clients requesting an amicable divorce

21
(iv)(b)(4): each client gives informed consent (confirmed in writing)

(a) Definition: an agreement to a proposed course of conduct after the


lawyer has communicated adequate information and explanation
about the material risks of and reasonably available alternatives to
the proposed course of conduct.
(b) Vicious Cycle: Sometimes this requires disclosure of the
confidences of Client B to Client A, which depends on Client Bs
consent (comment 19)!
(i) Solution: If Client B declines to allow the lawyer to disclose
the relevant confidences, the lawyer may not proceed with the
representation because the lawyer will not be able to obtain
informed consent from Client A. ==> Lawyer must withdraw
from at least one case.
(ii) Restatement Standard: provides more wiggle room in these
situations because prohibits revelation of confidences only if
there is a reasonable prospect that doing so will adversely
affect a material interest of the client or if the client has
instructed the lawyer not to use or disclose such information.
(c) Revocation: A client who has consented to a conflict may revoke
the consent (by firing the attorney).
(i) Solution: The lawyer may have to withdraw from representing
both clients, or he may be able to continue to represent one of
them.
b. Consentable Conflicts (of all types)
(i) Sophisticated Clients: In a practice in which the clients are large
institutional clients with general counsels advising them, nearly all
conflicts may be waived by consent.
(ii) Present v. Former Clients: If the conflict is between one present and
one former client, it is always consentable. (See Rule 1.9.)
(a) Reasoning: All of the first 3 requirements under Rule 1.7(b) are
satisfied!
c. Nonconsentable Conflicts: If a conflict is nonconsentable (because of one
of the first 3 requirements above), the lawyer must decline to accept
representation of one or more clients.
(i) If the nonconsentable conflict emerges after a lawyer has undertaken
the representation, the lawyer might be able to avoid the conflict by
withdrawing from the representation of one of the affected clients.
(Sometimes, however, the lawyer will have to give up both clients.)
4. Advance Waivers
a. Validity Dependent Upon
(i) how well the client understands the risks of the possible future
conflicts
(ii) the thoroughness and specificity of the lawyers disclosure of possible
future conflicts before the waiver was given

22
(iii) the clients experience with respect to the legal services being
provided and the nature of the conflicts that could arise
(iv)whether the client received independent legal advice before giving the
advance waiver
(v) whether the conflict that arises is one that can be solved by consent (if
it is nonconsentable, the waiver will not be valid as to that particularly
conflict)
b. Disfavored, and therefore only used with sophisticated clients
c. While a sophisticated client has the legal right to waive conflicts even
for subsequently-arisen related matters (where confidentiality concerns
abound), they most likely would not do so.
(i) Remember: Rule 1.9 (successive) conflicts can always be waived;
Rule 1.7 (concurrent) conflicts can only sometimes be waived.
5. Withdrawal and Disqualification
a. Withdrawal: If a concurrent conflict arises and the attorney cannot gain
exception to Rule 1.7(a) (by satisfying Rule 1.7(b)), the lawyer must
withdraw from the representation (see Rule 1.16).
(i) Prior Joint Representation: If a lawyer was jointly representing two
clients but it then becomes clear that there is direct adversity between
the two, the lawyer cannot drop just one client because that would
create a conflict (current vs. former client).
b. Disqualification: If the lawyer fails to withdraw, the opposing counsel may
file a motion to disqualify the lawyer from continuing the work.
6. Imputation of Concurrent Conflicts
a. General Rule (Rule 1.10(a))
(i) Policy: Exempts from imputation conflicts that do not involve client
loyalty or protection of confidences, but involve only a personal
interest of the layer (such as his political beliefs). (Comment 3)
(ii) Firm: includes lawyers in a law partnership, professional
corporation, sole proprietorship or other association, as well as lawyers
employed in a legal services organization or the legal department of
corporation or other organization. (Comment 1)
(a) Shared Offices: Lawyers who have separate practices but who
share office space may be considered to be a law firm for conflicts
purposes if their file management and communication is such that
one lawyer in a suite might have access to confidential information
about a matter being handled by another lawyer. (Comment 2)
(iii) Scope of Imputation: The interests of law clerks, paralegals, legal
secretaries, and other nonlawyer employees are not considered in a
conflicts analysis.
(a) Pre-Lawyer Employment: Rule 1.10(a) also does not apply to
conflicts that are presented by one lawyer because of work that that
person did before she became a lawyer. (Comment 4)
b. Exception to Rule (Rule 1.10(c)): Imputed conflicts may be waived by a
client affected by the client, subject to the restrictions articulated in Rule
1.7(b) (Rule 1.10(c)).

23
C. Conflicts Between Current Clients in Civil Litigation
1. Representation of Co-Plaintiffs or Co-Defendants
a. Possible Conflicts
(i) one client might have a claim against the other
(ii) if co-plaintiffs, the defendants assets may be too limited to satisfy
both claims
(iii) defendant may try to trade one clients claim off against the other
in making an imbalanced settlement offer
(iv)if co-defendants, each may wish to avoid liability by asserting that the
other is responsible
(v) if co-plaintiffs, might disagree as to what the remedy sought should be
(damages or injunctive relief)
b. Driver-Passenger Cases: Several cases have held that a driver and
passenger have sufficiently divergent interests that they may not be
represented by a single lawyer.
(i) Restatement Differs: allows waiver of such a conflict if informed
consent given (Restatement 128, comment d)
2. Representation of Economic Competitors in Unrelated Matters
a. No Conflict: Comment 6 to Rule 1.7 states that simultaneous
representation in unrelated matters of clients whose interests are only
economically adverse...does not ordinarily constitute a conflict of
interest.
3. Taking Inconsistent Legal Positions in Litigation (Positional Conflicts)
a. General Rule: Whether a positional conflict presents a serious conflict of
interest depends on the likelihood that one client would be materially
harms by a lawyers making an argument in another case that was contrary
to the clients interest.
(i) Considerations
(a) whether the decision in one case is likely to affect the decision in
the other
(b) whether the lawyer might be inclined to soft-pedal or otherwise
alter one or another argument to avoid affecting the other case
D. Conflicts in Nonlitigation Matters: Representation of Both Parties to a Transaction
1. General Rule: If there is an actual or potential conflict that is reasonably
apparent to the lawyer, he may not represent both parties without first
providing both with information about the possible downsides of the joint
representation and obtaining their consent.
a. This rule applies whenever a lawyer is approached by two clients seeking
legal assistance with a common goal!
b. Example: Two domestic partners wish to jointly purchase a home ==> no
conflict usually.
c. Example: A buyer and seller want to orchestrate a transaction ==>
possibility for conflict.
2. Confidences: Comment 31 to Rule 1.7 suggests that a lawyer usually should
NOT keep confidences form one joint client received from the other.
a. Reasoning: duty of loyalty owed to both parties

24
3. When Conflict Erupts: a lawyer may not continue to represent both (or all) of
the clients because that would involve the lawyer in suing one client on behalf
of another in an adversarial situation.
a. If the lawyer withdraws from representing one party, then that party
becomes a former client, and continued representation of the other party
is likely prohibited under Rule 1.9.
(i) Thus, the continued representation is only permitted with the consent
of the former client.
E. Joint Representation in Particular Practice Settings
1. Representation of Criminal Co-Defendants
a. Rules
(i) Model Rules Rule: Lawyers should decline to represent more than one
co-defendant in a criminal case. (Comment 23 to Rule 1.7, reasoning
that the potential for conflict of interest in representing multiple
defendants...is so grave)
(ii) ABA Rule: Except for bail hearings and other preliminary matters,
lawyers/firms should not take on more than one co-defendant (unless
there is no conflict, but this is rare). See Standards for Criminal
Justice: Prosecution Function and Defense Function, Standard 4-
3.5(c), Conflicts of Interest (3d. ed. 1993).
(iii) Restatement Rule: A lawyer may not represent criminal co-
defendants unless the clients give informed consent and it is
reasonably likely that the lawyer will be able to provide adequate
representation to...the clients. (Restatement 122, 129.)
b. Sixth Amendment: If a defendant is represented by a lawyer who has a
conflict of interest, he may challenge his conviction on the basis that he
was denied the effective assistance of counsel.
(i) Also, whenever a trial court improperly requires joint representation
over timely objection, reversal is automatic! (Holloway v. Arkansas,
1978)
(ii) If no timely Sixth Amendment objection is made, the defendant need
not demonstrate prejudice (as a result of the joint representation) but
only an adverse impact in the representation. (Cuyler v. Sullivan,
1980)
(a) Overturning of Convictions Less Likely: Under Mickens v. Taylor
(2002), if the trial judge is not aware of a conflict (and thus not
obligated to inquire), prejudice will be presumed only if the
conflict has significantly affected counsels performance, thereby
rendering the verdict unreliable.
c. Waiver of Conflict: Where there is a great risk of prejudice resulting from
a conflict, a judge may disqualify counsel from representing co-defendants
even if the co-defendants want to waive the conflict.
2. Representation of Family Members
a. Divorce: Some states allow lawyers to represent both spouses in an
uncontested divorce; others do not.

25
b. Estate Planning: Some authorities urge that joint representation of two
clients should include an agreement that information shared by one is not
to be held in confidence from the other.
(i) Florida Bar Opinion 95-4 (1997): In the absence of such an agreement,
a lawyers ethical obligation of confidentiality owed to Spouse 1
trumps his obligation of candidness to Spouse 2. Where an adversity
of interests arises as a result, the lawyer must withdraw and inform
Spouse 2 that a conflict of interest has arisen that precludes his
continued representation of both spouses.
3. Representation of Insurance Companies and Their Insureds
a. Situation: The lawyer has some obligations to the insurer, who pays the
fee and who will probably pay any damages awarded, and some
obligations to the insured, which obligations are sometimes in conflict.
b. Who is the Client?: If there is a conflict, the lawyer should act in the best
interests of the insured. If the insured is also the lawyers client, the
lawyer should try to act in the best interests of both clients.
(i) Rule 1.7 Conflict: If acceptance of payment from someone other than
the client presents a significant risk that the lawyers representation of
the client will be materially limited by the lawyers own interest in
accommodating the person paying the lawyers fee or by the lawyers
responsibilities to a payer who is also a co-client, there is a conflict
under Rule 1.7(a).
c. When to Withdraw
(i) When acting in the best interests of both clients is not possible
(ii) When the insurer and the insured disagree about whether to settle
d. Confidentiality: The lawyer is not permitted to reveal confidential
information from the insured person to the insurer.
4. Representation of a Class
a. Potential Conflicts
(i) a greater concern for the interests of the class representatives than for
the unnamed members of the class
(ii) a prior relationship with the named defendants in the class action
(iii) a greater concern for receiving a fee than for pursuing the class
claim
(iv)the settlement of claims by collusion rather than through a fair process
where class members interests are adequately represented
b. Rule 1.7 rules dont work well here (e.g., actual consent of all parties is a
practical impossibility).
(i) Thus, judges rarely apply Rule 1.7; other rules apply.
VI. Conflicts Involving Former Clients, Government Lawyers, and Judges
A. The Nature of Conflicts Between Present and Former Clients
1. How the Conflict May Harm a Former Client
a. Betray confidences of former client
b. Make adverse use of confidences learned from former client
c. Challenge work that the lawyer did on behalf of the former client

26
d. Engage in work that is in some other way disloyal to the former client or
at least causes the former client to feel betrayed
2. How the Conflict May Harm a Present Client
a. Be less zealous on behalf of the present client because of an obligation to a
former client
B. Duties to Former Clients
1. Limited Examples
a. Primary Duty: Protection of confidences
b. Avoidance of side-switching
c. Refraining from attacking the work the lawyer did for the former client
2. Application: Rule 1.9 provides protections for all former clients!
a. NB: A conflict between a former client and a present client triggers BOTH
Rules 1.7 (for protecting the present client) and 1.9 (for protecting the
former client)!!
3. Rule 1.9(a): Conflicts Between Former Client and Present/New Client
a. Lawyers: include summer associates!
b. General Rule: Representation of present/new client is prohibited IF...
(i) Substantial Relationship: matters are the same or substantially related,
AND
(a) Matter: anything that is the subject of representation (a litigation,
a transaction, a subject on which a client requests advice)
(b) Same: same parties, same lawsuit, same legal issues, same (or
overlapping) facts
(c) Substantially Related: asks whether the lawyer, in the course of
her work in the first matter, would normally have learned
information that could be used adversely to the former client in the
second (Comment 3)
(i) Objective Standard: involves a realistic appraisal of the
possibility that confidences had been disclosed in the one
matter which will be harmful to the client (that is, useful to the
adversary) in the other (not an inquiry into whether actual
confidences were disclosed) (Westinghouse Elec. Corp. v. Gulf
Oil Corp., 1978)
(a) Knowledge of Organizational Clients: General knowledge
of the clients policies and practices ordinarily will not
preclude a subsequent representation; on the other hand,
knowledge of specific facts gained in a prior representation
that are relevant to the matter in question (or knowledge of
how the organization handles certain types of cases in
litigation) ordinarily will preclude such a representation.
(Comment 3)
(i) Public Information or Obsolete Confidences: If the
confidential information learned from the former entity-
client has become public or is so out-of-date that it is
not relevant, it is unlikely that the new matter would be
found to be substantially related to the old one.

27
(b) Exam Tip: Focus on the facts and on what a lawyer might
have learned during the first matter that could be used
adversely to the former client in the second.
(ii) Example: A lawyer who has represented a businessperson and
learned extensive private financial information about that
person may not then represent that persons spouse in obtaining
a divorce. (Comment 3)
(ii) Material Adversity: present/new clients interests are materially
adverse
(a) Not defined in Model Rules or its Comments!
(i) Intent of Rule: to require consent from former client is use of
former clients confidences might harm him
(b) ABA Ethics Committee: urges that it be read as direct adversity
as in Rule 1.7
(c) Restatement: concludes that it is limited to potential harm to the
type of interests that the lawyer sought to advance on behalf of the
former client
c. Exception: if former client gives informed consent (confirmed in writing)
d. Policy: Its ALL about confidentiality!
4. Rule 1.9(b): Conflicts Between Present/New Client and Client of Lawyers
Old Firm
a. General Rule: Representation of present/new client is prohibited IF...
(i) Substantial Relationship: matters are the same or substantially related
AND
(ii) Material Adversity: present/new clients interests are materially
adverse AND
(iii) Acquisition of Material Confidences: the lawyer acquired
confidential information material to the matter.
(a) Subjective Standard: asks whether the lawyer actually acquired
material confidential information
(b) Legitimate Assumptions (per Comment 6)
(i) A lawyer who had management responsibility at a former firm
received confidential information about all firm matters.
(ii) A junior lawyer with no management duties had access to or
information about only the matters that she worked on or
perhaps only matters handled by her department.
(c) Rebuttable Presumption
(i) If your former colleague at your former firm did work for
Client A, and Client B asks you (at your new firm) to represent
him versus Client A, you can rebut the presumption that you
obtained confidential information about Client A while your
former colleague did work for her.
b. Exception: if former client gives informed consent (confirmed in writing)
c. Competing Policies
(i) protection of confidences shared by former clients
(ii) provision of relative freedom to clients in their choice of counsel

28
(iii) assurance of relative freedom to lawyers in moving from one firm
to another and in taking on new clients
d. Burden of Proof in Disqualification Suits: The firm whose disqualification
is sought should have the burden to prove that the lawyer who changed
firms does not possess confidential information that is material to the new
matter. (Comment 6)
5. Rule 1.9(c): Essentially instructs lawyers to protect confidences of former
clients (and clients of former firms).
a. Dual Prohibition
(i) revelation of confidences received from former clients
(ii) adverse use of such confidences
b. Exception: If the information received in confidence has since become
generally known, the lawyer need not keep it confidential.
c. Confidentiality Period: indefinite (during and after representation)
6. Waivability: Note that under both Rules 1.9(a) and (b), the conflict is always
waivable if the former client consents. There is no such thing as a
nonconsentable conflict involving a former client!
C. Distinguishing Present and Former Clients
1. Maintaining Contact: If a lawyer has not formally concluded representation of
the client, a court might still find that the client is a present one and that a
conflict with another client should be evaluated as a concurrent conflict.
2. Hot Potato Doctrine: If a lawyers withdrawal is premature and motivated by a
desire to dump Client A so that the lawyer can work for Client B, the lawyers
withdrawal represents a breach of the lawyers duty to Client A.
a. Bottom Line: No dumping clients to take on more lucrative clients!
b. When Withdrawal Is Appropriate (and wont trigger this Doctrine)
(i) if withdrawal occurs at natural end point in the representation
(ii) if client fires lawyer for reasons other than the impending conflict
(iii) if client triggers a conflict for the lawyer by taking some action
that was unforeseeable to the lawyer
(a) Withdrawal would make that client a former client.
(iv)if withdrawal occurs for some other good reason (e.g., to avoid
assisting a fraud)
3. Former In-House Counsel: If a lawyer worked on a matter (more than a trivial
amount) on behalf of his employer corporation, the corporation is his former
client as to that matter. However, the corporation mat not be the lawyers
former client as to matters that were going on during the lawyers employment
if the lawyer did not work on them and received no confidences regarding
those matters.
D. Particular Applications of Rule 1.9
1. Representing the Competitor of a Former Client: If a lawyer learned a great
deal about the operation of the former client and that information could be
used on behalf of a competitor to the disadvantage of that former client, the
lawyer could have a serious conflict.

29
a. Maritrans Case (1992): Whether a law firm can later represent
competitors of its former client is a matter that must be decided from case
to case and depends on a number of factors, including
(i) the extent to which the firm was involved in its former clients affairs
(the greater the involvement, the greater the danger that confidences
will be revealed)
E. Imputation of Former Client Conflicts to Affiliated Lawyers
1. Imputation of the Conflicts of a Departed Lawyer to His Former Firm
a. Rule 1.10(b): If an infected lawyer leaves a firm, the remaining lawyers
are no longer infected by the ex-lawyers conflict unless the new matter is
(i) substantially related to the old matter, and
(ii) one of the remaining lawyers possesses material confidences learned
as a result of the ex-lawyers previous work.
b. Waivability: Note that under Rule 1.10(c), an imputed conflict to the
former firms attorneys may be waived by the former client according to
Rule 1.7(b).
2. Imputation of the Conflicts of an Entering Lawyer Who is Infected
a. Rule 1.9(b): Representation of present/new client is prohibited IF...
(i) Substantial Relationship: matters are the same or substantially related
AND
(ii) Material Adversity: present/new clients interests are materially
adverse AND
(iii) Acquisition of Material Confidences: the lawyer acquired
confidential information material to the matter.
b. Special Screening Exception for Law Students: Law students who, before
joining the bar, worked at a firm that represented a client whose interests
are adverse to a client of their present firm, do not cause imputed conflicts
to arise at the new firm; instead,
(i) they are merely barred from personal participation in work for the
client of the new firm, and
(ii) they must be screened so that any confidential information they
learned at the former firm is not passed along to lawyers at the current
firm. (Comment 4)
c. Waivability: Note that under Rule 1.9 (b), the conflict is always waivable
if the former client consents. There is no such thing as a nonconsentable
conflict involving a former client!
F. Successive Conflicts of Present and Former Government Lawyers
1. Two Bases for Disqualification of Former Government Lawyers (now in
private practice)
a. Basis #1 (1.11(a)): precludes a former government lawyer from
representing a client in a matter in which the lawyer participated
personally and substantially while in government service.
(i) Applies only to the same matter as the previous work
(a) Matter
(i) Definition: any judicial proceeding, application, request for a
ruling or other determination, contract, claim, controversy,

30
investigation, charge, accusation, arrest or other particular
matter involving a specific party or parties, and any other
matter covered by the conflict of interest rules of the
appropriate government agency. (Rule 1.11(e))
(ii) Considerations: the extent to which the matters involve the
same basic facts or the same or related parties, and the time
elapsed (Comment 10)
(b) Personal and Substantial Participation
(i) Not defined in Comments.
(ii) Enlightenment from federal conflict of interest laws (from
which language was borrowed) (5 C.F.R. 2637.201(d)
(2004)):
(a) Personally: directly (including work done by subordinate
when directed by the former government lawyer)
(b) Substantially: of significance to the matter; forming the
basis for a reasonable appearance of such significance.
(c) The participation must be related to a particular matter
involving a specific party!
(ii) Waivability (1.11(a)): If the appropriate government agency gives its
informed consent (confirmed in writing) to the representation, its ok.
(iii) Imputation to Firm Attorneys (1.11(b)): If attorney is barred under
1.11(a), all other firm attorneys are barred as well.
(a) Exception: No imputation if
(i) attorney is screened
(ii) attorney receives no part of the fee (except if it is part of a
regular salary or partnership share)
(iii) written notice is promptly given to the appropriate
government agency (of the situation)
b. Basis #2 (1.11(c)): precludes a former government lawyer from
representing a client where the representation would involve use of
confidential government information about a person known to the lawyer
in a way that would materially disadvantage that person.
(i) Confidential Government Information: information
(a) that has been obtained under governmental authority
(b) and which the government is prohibited by law from disclosing to
the public or has a legal privilege not to disclose
(c) and which is not otherwise available to the public.
(ii) Waivability: None.
(iii) Imputation to Firm Attorneys (1.11(c)): No imputation arises if
(a) attorney is screened
(b) attorney receives no part of the fee (except if it is part of a regular
salary or partnership share)
2. Bases for Disqualification of Former Private Practice Lawyers (now in
government)
a. Rule 1.11(d)

31
(i) Rule 1.7 (prohibits lawyer from having concurrent conflicts = direct
adversity + substantial risk of material limitation in representation)
(ii) Rule 1.9(a) (prohibits lawyer from working on the same or a
substantially related matter in which that persons interests are
materially adverse to the interests of a former client unless that client
gives informed consent, confirmed in writing)
(iii) Rule 1.11(d) (prohibits lawyer from participating in a matter in
which the lawyer participated personally and substantially while in
nongovernmental employment)
(a) Waivability: If the appropriate government agency gives its
informed consent (confirmed in writing), its ok.
G. Conflicts Rules for Sitting Judges
1. General Rule: A judge must disqualify herself if her impartiality might
reasonably be questioned. (ABA Code Jud. Conduct, Canon 3E,
Disqualification (as amended 2003).) Situations include matters in which
a. she has a personal bias toward a party or lawyer
b. she knows about disputed facts in the case from personal experience
c. she has an economic interest in a case
d. she is related to a party, a lawyer, or someone with a substantial stake in
the case
e. she knows that she has received campaign contributions of more than a
specified amount from a party or lawyer
f. she has made a public statement that appears to commit her with respect to
an issue in the case
H. Conflicts Rules for Former Judges, Law Clerks, Arbitrators, and Mediators
1. General Rule (Rule 1.12(a)): A lawyer shall not represent anyone in
connection with a matter in which the lawyer participated personally and
substantially as a judge or other adjudicative officer or law clerk to such a
person or as an arbitrator, mediator or other third-party neutral.
a. Personal and Substantial Participation: same standard as under Rule
1.11 above.
b. Exception (Rule 1.12(a)): If all parties to the proceeding give informed
consent (confirmed in writing), its ok.
c. Imputation (Rule 1.12(c)): No imputation arises if former judge
(i) is timely screened from any participation in the matter, and
(ii) is apportioned no part of the fee therefrom, and
(iii) written notice is promptly given to the parties and any appropriate
tribunal (so that they can check whether the rule is observed)
2. Employment Negotiation (Rule 1.12(b)): prohibits lawyer-adjudicators from
negotiating for employment with any party or lawyer for a party to a matter in
which the adjudicator is participating personally and substantially
a. Exception: for recent law school graduates serving as law clerks (but
only after the lawyer has notified the judge or other adjudicative officer
for whom he is working) (Rule 1.12(b))
b. Post-Adjudication: adjudicator may engage in employment negotiation
(because Rule 1.12(b) is written only in present tense)

32
I. Conflicts Involving Prospective Clients
1. Prospective Client: any person who discusses with a lawyer the possibility
of forming a client-lawyer relationship (Rule 1.18(a))
a. Unilateral communication without the expectation of a possible
representation does not make someone a prospective client.
2. General Rule: Lawyer prohibited from using or revealing information learned
in the consultation. (Rule 1.18(b))
a. Regardless of the brevity of the contact (Comment 3)
b. Exception: except as Rule 1.9 would permit with respect to information of
a former client
3. Rule Disqualifying Lawyer from Representation: Lawyer subject to 1.18(b)
shall not represent a client with interests materially adverse to those of a
prospective client in the same or a substantially related matter if the lawyer
received information from the prospective client that could be significantly
harmful to that person in the matter. (Rule 1.18(c))
a. Imputation: If a lawyer is disqualified under 1.18(c), no lawyer in a firm
with which that lawyer is associated may knowingly undertake or continue
representation in such a matter. (Rule 1.18(c))
(i) Exception (Rule 1.18(d)): Representation is permissible if
(a) both the affected client and the prospective client have given
informed consent (confirmed in writing), OR
(b) the lawyer who received the information took reasonable measures
to avoid exposure to more disqualifying information than was
reasonably necessary to determine whether to represent the
prospective client, AND
(i) the disqualified lawyer is timely screened from any
participation in the matter, AND
(ii) the disqualified lawyer is apportioned no part of the fee
therefrom, AND
(iii) written notice is promptly given to the prospective client.
4. Advance Waiver: A lawyer may condition conversations with a prospective
client on the persons informed consent that no information disclosed during
the conversation will prohibit the lawyer from representing a different client in
the matter. The prospective client may also consent to the lawyers
subsequent use of the information received from the prospective client.
(Comment 5)
VII. Conflicts of Interest Between Lawyers and Clients
A. Legal Fees
1. Lawyer-Client Fee Contracts
a. General Rule of Reasonableness (Rule 1.5(a)): Requires that fees be
reasonable and provides a laundry list of criteria that lawyers should use to
evaluate the reasonableness.
(i) Unreasonable if lawyer has no records to document time spent
(ii) Improper fee if found to be disproportionate to services provided.
(iii) Telex (1979) Case

33
(a) Analysis: whether a contract is fair or works an unconscionable
hardship is determined with reference to the time when the contract
was made and cannot be resolved by hindsight.
(iv)In the Matter of Fordham (1996)
(a) Analysis: A fee is clearly excessive when, after a review of the
facts, a lawyer of ordinary prudence, experienced in the area of the
law involved, would be left with a definite and firm conviction that
the fee is substantially in excess of a reasonable fee.
(b) Analysis: It cannot be that an inexperienced lawyer is entitled to
charge three or four times as much as an experienced lawyer for
the same service.
(c) Analysis: While a safe harbor is available (entitling the attorney
to charge extra fees for time spent educating himself of the legal
area), the time spent must not be excessive.
(d) Analysis: The test is whether the fee charged is clearly excessive,
not whether the fee is accepted as valid or acquiesced in by the
client.
b. Communication about Fee Arrangements (Rule 1.5(b))
(i) Required Disclosures
(a) information about the scope of the representation
(b) the basis or rate of the fee and expenses
(c) NOT Required: an estimate of the expected total bill (but it would
be nice if a lawyer could provide a high and low prediction)
(ii) Required Writing
(a) only for contingent fees (see below)
(iii) Timing
(a) Disclosure must occur before or within a reasonable time after
commencing the representation.
(b) Emergency situations (e.g., client in jail) may justify delaying
disclosures relating to fees until after representation begins.
(iv)Modification of Fee Agreements
(a) Some courts hold that a lawyer may not simply notify a client of an
increase in the hourly rates charged.
(b) Other courts require client consent before a lawyer increases the
percentage of a settlement retained as a contingent fee.
(c) Principled Solution: Send a letter notifying of the increase and
request return of the letter to evidence clients consent to new rates.
2. The Dos and Donts of Billing
a. DONT bill for more hours than you actually work!
(i) Exception: rounding up to minimum billable time periods (e.g., 6
minutes) is ok.
b. DONT invent hours that werent really worked!
(i) Federal Mail Fraud: If a lawyer inflates his hours and then transmits
the resulting bill to a client, he may be charged with federal mail fraud
or other crimes.
c. DONT bill for overhead!

34
(i) These costs (e.g., costs of running a library, purchasing malpractice
insurance, paying for office space, heating or air conditioning) should
be accounted for in lawyers hourly rate.
d. DONT mark up costs!
(i) A lawyer may not bill the client more than the actual cost of the
service (e.g., photocopies, messenger service, postage) to the lawyer.
e. DONT double-bill!
(i) A lawyer may not bill two clients for one period of time.
(ii) Ethical Solution: Bill each client for half of the time, instead!
f. DONT bill a second client for recycled work!
g. DONT exploit an hourly fee arrangement by using wasteful procedures!
(Rule 1.5, Comment 5)
(i) Continuous toil on or overstaffing a project for the purpose of
churning out hours is not properly considered earning ones fees.
(ABA Opinion 93-379)
h. DONT bill clients or the firm for personal expenses or inflated expense
receipts!
i. DONT bill by the hour at lawyer rates for performance of administrative
services!
(i) Delegate such work to staff nonlawyer staff.
j. DONT bill for time spent drafting the bill! (says an ambiguous 1999
opinion of the Indiana Supreme Court)
3. Contingent Fees
a. Policy: allow access to justice for people who are not wealthy
b. Reasonableness: Required (per Rule 1.5(a))
(i) No maximum percentage is specified by the Rules.
(ii) Normally 1/3
(a) higher percentages may be legitimate if possibility for recovery is
remote
(b) lower percentages are urged where recovery is all but guaranteed
(and not much work is necessary)
c. Exception (Rule 1.5(d)): Contingent fees are not permissible in criminal
defense (Rule 1.5(d)(2)) and certain domestic relations cases (Rule 1.5(d)
(1)).
(i) Policy: permitting contingent fees in these cases might tempt the
lawyer to allow the client to give false testimony or to discourage the
client from pleading guilty
(ii) Doesnt apply to collection of past due alimony or child support cases.
d. Pre-Representation Writing Required (Rule 1.5(c))
(i) must be signed by the client
(ii) must state the method by which the fee is to be determined, including
(a) the percentages applicable in the event of settlement, trial or
appeal,
(b) litigation and other expenses to be deducted from recovery, and
(c) whether such expenses are to be deducted before or after the
contingent fee is calculated.

35
(i) Not Required: deduction of expenses before calculation of
contingent fee
(iii) must explain which expenses the client must pay even if she gets
no recovery
e. Post-Representation Writing Required (Rule 1.5(c))
(i) must state outcome of the matter
(ii) if recovery, must explain what fee and expenses were charged and how
they were calculated
(iii) must show remittance to the client
f. Proposal by Manhattan Institute
(i) If an offer is on the table and then the client seeks out a lawyer, who
advises client to accept the offer, lawyer may not take a contingent fee
from that settlement.
(ii) If an offer is made within 60 days of plaintiffs demand, and it is
rejected, plaintiffs lawyer can only get a percentage of the net
recovery (in excess of the offer).
4. Forbidden and Restricted Fee and Expense Arrangements
a. Purchasing Legal Claims: Rule 1.8(i) prohibits lawyers from acquiring an
interest in litigation on behalf of a client, except for permitted liens and
contingent fees. (A lawyer may not purchase an interest in a claim on
which the lawyer is representing the client.)
b. Financial Assistance to Clients: Rule 1.8(e) prohibits lawyers from
providing financial assistance to a client.
(i) Exceptions: advancement of court costs and expenses of litigation
(which may be repaid); payment of court costs and expenses of
litigation in pro bono representation.
(a) Example: costs of medical examinations or other costs to obtain
evidence; transportation costs to and from courts
(ii) Policy: The Bar does not want lawyers to compete for clients by
offering extravagant perks.
(iii) Policy: Insulates lawyer from feeling guilted into providing
financial assistance to needy clients.
c. Publication Rights: Rule 1.8(d) prohibits lawyers from contracting for
literary (book/media) rights as payment for legal services before
conclusion of the legal representation.
(i) After representation is concluded, such agreements may be made.
(ii) Policy: A lawyer in such a situation might be tempted to do things that
would be bad for the client and good for the book/movie.
d. Advance Payment of Fees and Nonrefundable Retainers
(i) Retainers (Rule 1.16(d)): If a lawyer charges a retainer, he must
deposit the sum in the client trust account, withdraw portions only as
they are earned, and return any unearned portion to the client.
(a) Nonrefundable Retainers: NO SUCH THING (disfavored by
public policy as inconsistent with a lawyers fiduciary obligations
to his client)

36
(ii) Lump-Sum Advance Payments: secures the lawyers availability for a
period of time or for a particular task
(a) treated as earned when paid
(i) Policy: because lawyer may have to forgo other obligations to
maintain his availability
5. Fee Disputes & Fee Collection
a. Fee Arbitration: Lawyers are required to comply with mandatory
mediation or arbitration processes for resolution of fee disputes as required
by state law. (Rule 1.5, Comment 9)
b. Collection of Fees
(i) Options to Pursue Payment
(a) file lawsuit
(b) use a collection agency
(ii) NOT Options to Pursue Payment
(a) acts of harassment against a debtor or making a frivolous claim
(b) retention of documents or unearned fees as leverage to secure
payment of fees owed
(i) Remember: Documents may only be withheld from the client
to the extent they have not been paid for and to the extent that
such withholding will not unreasonably harm the client.
(c) false or misleading statements about the fee claim
(d) threats to reveal confidential information
c. Liens on Client Property
(i) Rule 1.8(i): permits a lawyer to acquire a lien authorized by law to
secure the lawyers fee or expenses
(a) Terms governed by Rule 1.8(a), as a lien contract constitutes a
business transaction with a client (see below for more)
d. Fees Owed to a Lawyer Who Withdraws or is Fired Before Completion
(i) Withdrawal
(a) General Rule: whether the lawyer is entitled to payment for the
work done depends on the reason she withdrew
(i) If withdrawal is valid, lawyer is entitled to payment on a
quantum meruit basis.
(ii) If the lawyer withdraws without good cause, the lawyer is
breaching the contract and may forfeit her right to recover any
fees for the work done.
(b) Valid Reasons to Withdraw (Rule 1.16(b) all permissive)
(i) (1): if withdrawal can be accomplished without material
adverse effect on the clients interests;
(ii) (2): if the client persists in acting in a way the lawyer
reasonably believes is criminal or fraudulent;
(iii) (3): if the client has used the lawyers services to perpetrate
a crime or fraud;
(iv)(4): if the client persists in acting in ways the lawyer considers
repugnant or with which the lawyer fundamentally disagrees;

37
(v) (5): if the client fails substantially to fulfill an obligation to the
lawyer regarding the lawyers services and has been given
reasonable warning that the lawyer will withdraw if the
obligation goes unfulfilled;
(a) Ex.: client breaches contract with lawyer (e.g., by failing to
cooperate in representation)
(vi)(6): if the representation would unreasonably burden the lawyer
financially (or has been rendered unreasonably difficult by the
client)
(vii) (7): if other good cause for withdrawal exists.
6. Dividing Fees With Other Firms or With Nonlawyers
a. Division of Fees Between Lawyers Not in the Same Firm
(i) Rule 1.5(e): permits sharing fees (including fees for referral) if
(a) division of fee is proportionate to services performed or
responsibility assumed (or, in the case of a referral if, if the
referring attorney takes on financial and ethical responsibility for
the representation), and
(b) the client agrees to the arrangement, and
(c) the agreement is confirmed in writing, and
(d) the total fee is reasonable.
b. Sharing Fees with Nonlawyers
(i) General Rule (Rule 5.4(a)): prohibits fee-splitting with nonlawyers
(a) Exceptions: nonlawyer employees may be included in a
compensation or retirement plan, even though the plan is based in
whole or in part on a profit-sharing arrangement; nonlawyers may
also be paid salaries and bonuses (but they may not receive a share
of annual profits).
(ii) Runners: Lawyers may not pay referral fees to nonlawyers who send
them clients, except that they may pay lawyer referral services for
making information available about their practices.
7. Payment of Fee By a Third Party
a. Rule 1.8(f): Permits a third party to pay a lawyers fee, but only if
(i) the client consents after being advised,
(ii) the third party does not direct the lawyers decisions or otherwise
interfere in the representation, and
(iii) the lawyer avoids sharing with the third person any confidences
learned in the course of the representation.
B. Conflicts With Lawyers Personal or Business Interests
1. In General
a. Rule 1.7(a)(2): A lawyer has a conflict of interest if there is a significant
risk that the representation of one or more clients will be materially
limited by a personal interest of the lawyer.
b. Rule 1.7(b): If such a conflict arises, a lawyer must not represent the client
unless all of the conditions in Rule 1.7(b) are met.
2. Business Transactions Between Lawyer and Client

38
a. Rule 1.8(a): does not prohibit such transactions but strongly discourages
them
(i) Requirements: If a transaction is entered into,
(a) the client is entitled to
(i) fair terms,
(ii) a clear explanation of the terms in writing, and
(iii) written encouragement and opportunity to seek the advice
of an independent lawyer before the client agrees, and
(b) the client must give informed consent (confirmed in writing) to the
terms of the deal and to the lawyers role in the deal.
b. Policy: A lawyers legal skill and training, together with the relationship of
trust and confidence between lawyer and client, create the possibility of
overreaching when the lawyer participates in a transaction with a client.
(Comment 1 to Rule 1.8)
c. Application: applies to all contracts for goods and services regardless of
whether the lawyer is the buyer or seller and regardless of whether or not
they are related to the matter on which the lawyer represents the client.
(i) Two Exceptions
(a) contracts for legal services (unless part or all of the fee is to be
paid by the transfer of property to the lawyer)
(b) contracts in which the client sells the lawyer some product or
service in the ordinary course of business
(ii) Example: A lawyer may be paid in the form of stock or stock options,
but this type of transaction requires compliance with Rule 1.8.
3. Gifts from Clients (Rule 1.8(c))
a. prohibits lawyers from soliciting substantial gifts or bequests from a client
b. prohibits lawyers from preparing for a client any instrument (such as a
will) giving a gift to the lawyer or his relatives
(i) Exception (for both prohibitions): unless the lawyer is a close relative
(spouse, child, grandchild, parent, grandparent or other relative or
individual with whom the lawyer or the client maintains a close,
familial relationship).
c. allows lawyers to receive unsolicited gifts from clients
(i) Exception: if the gift is too large, the client could later sue for its
return.
d. Policy: Some lawyers might take advantage of their clients trust and
appreciation by persuading clients to give them property.
4. Sexual Relationships with Clients
a. General Rule: If a lawyer coerces a client into a sexual relationship (e.g.,
in lieu of paying a fee), the relationship is improper because it is
exploitative.
b. Rule 1.8(j): prohibits sexual relationships between lawyers and their
clients
(i) Exception: unless the sexual relationship existed before the lawyer-
client relationship began
5. Intimate or Family Relationships With Adverse Lawyers

39
a. Rule 1.7, Comment 11: If the relationship between two lawyers is that of
parent, child, sibling or spouse, they normally may not represent clients
who are adversaries unless the clients have been informed of the
relationship, advised of the possible problems, and given informed
consent.
(i) No Imputation: because considered a personal conflict of the lawyer
6. Imputation of Personal Interest Conflicts to Other Lawyers in a Firm
a. Financial Interest Conflicts
(i) Rule 1.8(k): Financial interest conflicts of one lawyer in a firm are
imputed to all other lawyers in the firm.
b. General Rule on Imputation of Conflicts With a Lawyers Interests
(i) Rule 1.10(a): If a personal interest conflict does not present a
significant risk of materially limiting the representation of the client by
the remaining lawyers in the firm, the conflict is not imputed to other
lawyers in the firm.
(a) Note: One must evaluate each such conflict on its facts rather than
attempting to make categorical distinctions.
C. Lawyer as Custodian of Client Property and Documents
1. Client Trust Accounts
a. Rule 1.15(a): Client funds possessed by the lawyer must be kept in a
separate account (separate from the lawyers own property) maintained in
the state where the lawyers office is situated, or elsewhere with the
consent of the client or third person.
(i) The lawyer must also keep complete records of the funds or other
property (deposits and withdrawals) for a recommended (by the ABA)
period of 5 years.
b. Comment 1: Lawyers may keep funds belonging to multiple clients in a
single account, so long as the account is separate from funds belonging to
the lawyer.
c. Lawyers may not deposit their own funds into a client trust account in
order to hide it from their own creditors.
2. Responsibility for Client Property
a. Prompt Delivery of Funds or Property
(i) Rule 1.15(d): When a lawyer receives a settlement check or other
funds that should be paid (at least in part) to a client, the lawyer is
obliged to notify the client and to make prompt payment of all funds
due to the client.
(a) If the client asks, the lawyer must provide a record of the amount
received and of how much was paid to whom.
b. Disputes About Money or Property in Lawyers Possession
(i) Rule 1.15(c): If there is a dispute about the amount of the fee, the
lawyer is to distribute undisputed portions of the settlement and keep
the disputed portion in the client trust account.
c. Lawyers Responsibilities to Clients Creditors

40
(i) Unless the creditor has a legitimate claim to the particular funds in the
lawyers possession (via an order against the lawyer to surrender the
funds), the lawyers duty is to his client, not to the third party.
3. Administering Estates and Trusts
a. General Rule: A lawyer may accept appointment as executor of a clients
estate.
(i) Rule 1.8(c): does not prohibit receipt of executors fees because the
payment would be for services rendered (rather than as a gift).
VIII. Lawyers Duties to Courts, Adversaries and Others
A. Pre-Filing of Complaint: Investigation Before Filing a Complaint
1. Required Investigation By Lawyers Filing Civil Cases
a. General Rule: Lawyers may not file frivolous lawsuits or make frivolous
arguments.
(i) Frivolous: on the basis of mere speculation that a prospective
defendant has done something wrong
(a) Exception: If SOL is about to expire, its more ok to file a
complaint on basis of plaintiffs outrageous statements.
(ii) Exception: Criminal defense attorneys may defend their clients so as to
require that every element of a case be established.
b. Comment 2: The facts need not be fully substantiated before suit is filed
(as discovery may be required to develop vital evidence).
2. Penalties for Bringing Unsubstantiated Lawsuits
a. Under Rule 3.1: disciplinary action by state bar
b. Under FRCP 11: sanctions imposed by judge in civil action (which can
result in nonmonetary directives or monetary sanctions against a lawyer or
party)
(i) Safe Harbor Provision: If opposing party files suit for violation of
FRCP 11, lawyer has 21 days to withdraw the allegedly frivolous
pleading and will suffer no sanction for so doing.
(ii) A lawyer who initiates a federal court lawsuit in good faith and later
finds out (e.g., through discovery) that the lawsuit is groundless may
be subject to sanctions.
c. Liability for Malicious Prosecution
(i) Elements of COA
(a) Plaintiff won previous suit (in which she was defendant).
(b) Prior suit was brought without probable cause.
(c) Prior suit was brought with malice (a motivation other than
obtaining a proper adjudication of the case).
(d) Plaintiff was injured despite having won the prior suit.
(ii) Defense: Defendant in this proceeding can prove probable cause for
the previous suit if the lawyer had a reasonable belief that facts
could be established and that, under those facts, the client had a valid
claim.
B. Post-Filing of Complaint: Truth and Falsity in Litigation
1. Rule 3.3(a)
a. (1): Falsities by Lawyers

41
(i) Bars false statements to courts by lawyers themselves, and requires
lawyers to correct any false statements they previously made
(ii) Tribunals include administrative agencies.
b. (2): Adverse Authority
(i) Lawyers must affirmatively disclose directly adverse law in the
controlling jurisdiction if the opponent doesnt do so.
c. (3): Falsities by Clients
(i) A lawyer who knows that his client or other witness is going to lie to
the court may not allow the witness to do so.
(a) Applies to trial testimony, depositions, and other testimony related
to adjudication. (Comment 1)
(ii) If the witness does lie, the lawyer must call on the witness to correct
the lie, and if he wont, the lawyer must disclose the lie.
(a) Applies to both civil and criminal matters.
(i) Criminal Narrative Compromise: allows criminal defendant to
lie to a jury by telling a story rather than responding to direct
questions posed by the lawyer; this is prohibited under the
Model Rules
(b) Restatement Solution: Lie should be disclosed to the prosecutor (if
a criminal case), who may back your client into his lie but may not
refer to your disclosure in so doing (under Restatement 120).
(iii) Reasonable Belief of Falsity (but Uncertain)
(a) Civil: the lawyer may refuse to offer the evidence (but is not
required to do so).
(b) Criminal: the lawyer must allow the defendant to testify
2. Rule 3.3(b): Criminal/Fraudulent Conduct by Clients
a. A lawyer must take reasonable remedial measures if a person intends to
engage, is engaging or has engaged in criminal or fraudulent conduct in
connection with a case before a tribunal.
(i) Remedial Measures
(a) Try to talk the person into recanting
(b) Warn person you will have to disclose lies and person will be
found guilty of perjury
(ii) Person: If OP (or OPs witness) lies on the stand, even if the lie
helps your clients case, you still need to take remedial measures.
(a) Contrast Restatement 120: Lawyers have no obligation to correct
an opposing partys lies.
(iii) Criminal or Fraudulent Conduct: A clients intent to lie on the
witness stand constitutes criminal or fraudulent conduct (because
perjury is a crime).
(a) Client Half-Truths
(i) No Rule directly addresses half-truths.
(a) Very little case law speaks to whether lawyers can be
disciplined because they or their witnesses misled courts
with partially true but deceptive testimony.

42
(b) Lawyers are never or rarely disciplined or sanctioned for
the half-truths of their testifying clients or witnesses.
(ii) All that being said, to the extent the half-truth works a fraud on
the tribunal, Rule 3.3(b) may require the lawyer to take
reasonable remedial measures.
3. Rule 3.3(c)
a. Duration of Duty: until conclusion of proceeding
(i) Deduction: Thus, if you learn that your client lied to the court after the
proceeding is over, you have no duty to report the lies.
b. Overrides confidentiality duties under Rule 1.6
4. Rule 3.3(d): Ex Parte Proceedings
a. Lawyer has a duty to tell the court about adverse facts as well as adverse
law.
5. Rule 8.4(c): It is professional misconduct for a lawyer to...engage in conduct
involving dishonesty, fraud, deceit or misrepresentation.
a. Applies to all conduct by lawyers, including conduct before tribunals.
b. Broader than duty under Rule 3.3(a)(1).
c. Comment 2: Lawyers will only be disciplined if the things they do reflect
on their fitness to practice law.
d. Opposing Counsel: While lawyers may not lie to OP Counsel, they have
no duty to answer their impromptu questions.
e. Identity Dishonesty: Does not apply to misrepresentations solely as to
identity or purpose and solely for evidence-gathering purposes. (See, e.g.,
Apple Corps, Ltd. v. International Collectors Society, D.N.J., 1998.)
C. Concealment of Physical Evidence and Documents
1. Rule 3.4(a)
a. Unlawfully: not defined in the Rules, but clearly requires another (state)
law in order for Rule 3.4 to make concealment or destruction of evidence a
violation!
(i) ABA Annotated Model Rules: defines unlawful as only if the
lawyer already has some obligation to disclose it, such as if the
evidence is the subject of a discovery request to which no objection
has been made, or is the fruit or instrumentality of a crime and the
lawyer is required by law to turn it over to law enforcement officials.
(ii) Obstruction of Justice: If the conduct at issue would violate a criminal
obstruction of justice statute, the destruction or concealment is
unlawful.
(iii) Contempt of Court: If the conduct violates a court order, it is
unlawful because it is a contempt of court.
b. Destruction: Just as a lawyer may not conceal incriminating physical
evidence, the lawyer may not destroy such evidence or counsel the client
to do so.
c. General Rules
(i) If a client tells a lawyer about the location of evidence, the lawyer may
inspect the evidence but should not disturb it or move it unless doing
so is necessary to examine or test the evidence.

43
(a) If the lawyer merely inspects the evidence without disturbing it,
the lawyers knowledge of its location remains privileged.
(b) In many states, a lawyer may take temporary possession of
physical evidence of client crimes for the purpose of conducting a
limited examination that will not alter or destroy material
characteristics of the evidence...Applicable law may require the
lawyer to turn the evidence over to the police or other prosecuting
authority...
(i) Not all jurisdictions require lawyers to contact prosecutors
about physical evidence in their possession.
(ii) If a client delivers physical evidence of a crime to a lawyer, the lawyer
must turn the evidence over to the law enforcement authorities within
a reasonable period of time at least in some jurisdictions. (This is
the safest thing for you to do as a lawyer and thus the prescribed
solution!)
(a) This rule applies to documents as well as to other physical
evidence.
(b) A prosecutor who receives evidence of a crime from the lawyer for
a suspect should take steps to avoid revealing to a jury the fact that
the incriminating evidence came from the defendants lawyer.
(i) However, if the lawyer received the evidence not from his
client but from a witness to the crime (and friend of the client),
the prosecutor could put the lawyer on the stand to give
testimony of where the evidence came from. (Confidentiality
and privilege will not shield the lawyer, as the testimony
doesnt involve communications with the client.)
(c) Putting It Back: A risk-analysis regarding the likelihood you would
be charged with concealing the evidence in violation of 3.4(a).
(i) Giving It Back: This risk escalates if you give the evidence
back to the client ==> dont do that!
d. Rule 3.4 for Criminal Matters
(i) If a lawyer has no knowledge that a violation of law has been
committed and no criminal investigation is foreseeable, a lawyer has
no duty to turn evidence over to a prosecutor.
(ii) In some states, the lawyers duty not to conceal tangible evidence
takes effect as soon as the lawyer believes that an official investigation
is about to be instituted. In other states, it does not begin until an
investigation has actually started.
e. Rule 3.4 for Civil Matters
(i) Soon after a civil case is commenced, a lawyer may have a duty under
the pertinent rules of procedure to turn over some information to the
opposing party, even in the absence of a discovery request (e.g.,
evidence planned to present at trial).
(ii) Some state laws require the preservation of business records for
specified periods of time even if no dispute is on the horizon. In
general, when a lawsuit is pending or foreseeable, individuals and

44
businesses have more stringent duties to protect and eventually to
disclose relevant material.
(iii) Where no specific record preservation statute applies but a lawyer
has some reason to believe that wrongdoing has occurred, state law
varies as to when the duty to preserve evidence kicks in.
(a) Once a duty to preserve documents applies, relevant records and
objects should be retained even if they could otherwise routinely
be destroyed.
f. Bottom Line: A lawyer should never assume that documents that are or
could be pertinent to a civil lawsuit may be concealed or destroyed, even if
a suit has not yet been filed.
2. Rule 3.4(b): A lawyer shall not...falsify evidence, counsel or assist a witness
to testify falsely, or offer an inducement to a witness that is prohibited by
law...
a. Preparing Witnesses to Testify: only a few courts have limited what
lawyers can do in depositions (e.g., objecting to questions to hint to a
client to be careful in answering).
(i) Restatement: more explicitly allows coaching that does not induce
false evidence. (116, comment b)
3. Rule 3.4(d)
a. No Frivolous Discovery Requests!
b. Make Reasonably Diligent Efforts to Comply With Proper Discovery
Requests!
4. Rule 4.4: Respect for Rights of Third Persons
a. Fax Problem: Under Rule 4.4(b), A lawyer who receives a document
relating to the representation of the lawyers client and knows or
reasonably should know that the document was inadvertently sent shall
promptly notify the sender.
D. Communications with Lawyers and Third Parties
1. Rule 4.1(a): No false statements of material fact or law to a third person!
a. Compare to Rule 3.3: bars false statements to tribunals
b. If Client Lies to Third Parties: Lawyer should take client aside and urge
the client to be truthful, but if lawyer fails to do so, no rule requires the
lawyer to correct the falsehood.
(i) Perpetration of a Fraud: A lawyer could be disciplined, however, for
sitting by silently if the client perpetrates a fraud.
c. Negotiations: The text of Rule 4.1 does not allow lawyers to lie for the
purpose of bargaining.
(i) Exception for Puffing: In negotiation, certain types of statements
ordinarily are not taken as statements of material fact. Estimates of
price or value placed on the subject of the transaction and a partys
intentions as to an acceptable settlement of a claim are ordinarily in
this category, and so is the existence of an undisclosed principal except
where nondisclosure of the principle would constitute fraud.
(Comment 2)

45
d. Rule 4.1(b): imposes on lawyers a limited duty to make affirmative
disclosures to others when necessary to avoid assisting a criminal or
fraudulent act by a client.
(i) However, the duty to disclose is subordinate to the duty to protect
confidential information.
2. Rule 4.2
a. General Rule: Dont communicate with opposing parties if they are
represented by attorneys!
(i) If a lawyer starts a conversation with another person erroneously
believing that the person is unrepresented, he must end the
conversation upon learning that the person has a lawyer.
(ii) The lawyer may communicate with the person about other subjects,
even closely related subjects, if they involve a different matter.
(iii) A lawyer may not circumvent this rule by directing someone else
to contact the represented person.
(iv)Two people who are represented by lawyers may talk with one another
without their lawyers permission; however, if a lawyer tells her client
to call the opposing party for some particular purpose, the lawyer
might be violating Rule 4.2 through the acts of another.
b. Policy: concern for overreaching (a represented person may make
disclosures or concessions that his lawyer would have counseled him to
avoid)
c. Application: whether the lawyer OR the party initiates the communication
d. Exception: If the represented client contacts a second lawyer to obtain a
second opinion or to explore changing lawyers, the second lawyer may
talk to the represented client. (Restatement 99, comment c)
e. Regarding Corporate Employees
(i) Comment 7 to Rule 4.2: states that an attorney may not speak ex parte
to 3 categories of employees:
(a) those who supervise or direct or regularly consult with the
organizations lawyer concerning the matter,
(b) those who have authority to obligate the organization with respect
to the matter
(c) those whose act or omission in connection with the matter may be
imputed to the organization for purposes of civil or criminal
liability (i.e., who are wrapped up in the litigation because they
acted in the scope of employment and caused damage).
(ii) Former Employees: Consent of the organizations lawyer is not
required for communication with a former employee. (Comment 7 to
Rule 4.2)
3. Rule 4.3
a. General Rules
(i) The lawyer should not mislead the person into thinking that the lawyer
doesnt represent a client.
(a) But the lawyer has no affirmative duty to clarify her role unless she
knows or should know that the other person is confused.

46
(ii) The lawyer should not state or imply that the lawyer is looking out for
the interests of both the client and the unrepresented person.
(iii) A lawyer may not ask a person other than the lawyers own client
to refrain from voluntarily giving relevant information to another
party unless the unrepresented person is a relative, employee, or
other agent of the lawyers client and the lawyer believes that the
witness interests wont be harmed by clamming up.
b. Comment 2: The lawyer may still negotiate a deal or settlement with an
unrepresented person.
(i) The lawyer may even prepare the relevant documents for both parties
to sign.
(ii) The lawyer may also inform an unrepresented party what the law is, so
long as he does not give legal advice.
(a) Example: Landlord-Tenant dispute, tell LL that he will owe treble
damages if he doesnt readmit the tenant to the apartment.
(b) Example: Child Support dispute, tell OP that he will owe $x under
the Child Support Guidelines.
c. Persons Protected: persons with whom a client has a dispute; potential
witnesses; and experts
(i) Includes employees when client is their corporation (employer)
(a) The lawyer should advise the employee at the beginning of the
interview that he represents the organization rather than the
individual.
(b) The lawyer should explain that she may share any information that
the employee reveals with officers of the corporation or with law
enforcement personnel or other third parties, if such disclosure is
in the interest of the organization.
(c) The lawyer should periodically assess whether there is a risk of
criminal liability for the employee being interviewed or others, and
if so, whether she should advise the employee of the option of
hiring his own lawyer and of having his lawyer present during the
interview.
IX. The Provision of Legal Services
A. Unauthorized Practice: Nearly every state has a statute barring nonlawyers from
practicing law or a court doctrine permitting the state bar association or state
officials to bring suits to enjoin unauthorized practice.
1. What does it mean to practice law?
a. DEs description:
giving legal advice
drafting legal documents or pleadings
appearing as legal counsel
performing duties traditionally done by lawyers
Nonexample: Boat accessories salesperson informing you
what equipment the law requires you to have on board.
2. Policy Ramifications

47
a. Positive: Protects clients by ensuring they receive legal advice only from
those qualified (certified) to provide it.
b. Negative: Makes it virtually impossible for low- and moderate-income
persons to obtain essential legal advice (because operates to monopolize
the industry).
B. Lay Advocates
1. Domains Where Lay Advocacy Permitted
a. Social Security Administration
b. Department of Homeland Security
2. Blurry Domains (Specialized Industries)
a. Tax: Accountants who prepare tax returns (which requires them to
interpret a complex statute); permitted to do this by federal legislation (as
Enrolled Agents with the IRS)
b. Real Estate: Realtors who prepare deeds to property and manage the
execution of closing documents.
3. Paraprofessionals: Lawyers are allowed to hire and train nonlawyers as
paraprofessionals, who may help clients with routine legal problems, such as
filling out forms, and may offer routine advice.
a. However, the employing lawyer must supervise these employees and take
responsibility for their advice and other work. (If the employee violates a
rule of professional responsibility, the supervising lawyer may be
disciplined for the violation.)
b. Lawyers may not assist a nonlawyer in practicing law.
C. Other Legal Restrictions on the Free Market for Legal Services

1. Advertising and Solicitation


a. General Advertising
(i) Bates v. State Bar of Arizona (1977): held that a state could prohibit
false advertising by lawyers, just as it could prohibit false advertising
by any type of merchant.
b. In Person Solicitation
(i) In 1978, the Court held that although a state may not prohibit truthful
advertising aimed at the general public, it may ban in-person
solicitations by lawyers.
(a) Policies
(i) to reduce the likelihood of overreaching and the exertion of
undue influence on lay persons
(ii) to protect the privacy of individuals
(iii) to avoid situations where the lawyers exercise of judgment
on behalf of the client will be clouded by his own pecuniary
self-interest.
c. Direct-mail Solicitation
(i) The Court has invited states to require mailings to bear a label
identifying them as advertisements or to direct the recipient how to
report inaccurate or misleading letters.

48
d. The Ethics Rules
(i) Rule 7.2(a): permits advertising through written, recorded or
electronic communication, including public media.
(ii) Rule 7.1: prohibits a lawyer from making a false or misleading
communication about the lawyer or the lawyers services and defines
such a communication to include both one that includes a material
misrepresentation and one that omits a fact necessary to make the
statement considered as a whole not materially misleading.
(iii) Rule 7.3: prohibits in-person, live telephone or real-time
electronic contact to solicit professional employment when a
significant motive for the lawyers doing so is the lawyers pecuniary
gain, unless the person contacted is a lawyer or has a family, close
personal, or prior professional relationship with the lawyer.
(a) Significant Motive Clause: permits real-time solicitation by legal
aid lawyers who do not charge fees for their services.
(b) Advertising Material: Does not bar mailed solicitation, but it
requires the sender to include the words Advertising Material on
the outside of the envelope.
2. Interstate Law Practice
a. Interstate Advertising: In the name of consumer protection, state bars
protect their lawyers from outside competition and regard poaching by
out-of-state lawyers (including mere advertising) as unauthorized practice,
just as if an out-of-state lawyer were not a lawyer at all.

b. Interstate Practice
(i) Rule 5.5: permits a lawyer otherwise in good standing to perform legal
services in another host state on a temporary basis if any one of four
factors applies:
(a) The lawyer is affiliated with another lawyer licensed in the host
state who actively participates in the matter.
(b) The lawyer is preparing for pending or potential litigation and is
admitted to appear in the proceedings or reasonably expects to be
admitted.
(c) The lawyers work in the host state is incident to a pending or
potential alternative-dispute-resolution proceeding and is
reasonably related to the lawyers practice in a state in which he or
she is admitted.
(i) Reasonably Related: situations in which the out-of-state
lawyer has recognized expertise developed through the
regular practice of law on behalf of clients in matters involving
a particular body of federal, nationally-uniform, foreign, or
international law. (Comment 14 to Rule 5.5)
(d) The lawyers work arises out of practice in a jurisdiction in which
the lawyer is licensed (e.g., work for a home-state client, work on a

49
matter with significant connection to the home state, and work in
the lawyers area of concentration).
(ii) Multijurisdictional Practice Amendments to the Rules
(a) 4 Safe-Harbors under Rule 5.5(c): Lawyers may leave their
jurisdiction to continue their representation of a client if...
(i) Local Association: they associate with a lawyer in the host
jurisdiction, or
(ii) Pro Hac Vice: they seek to be admitted in the host jurisdiction
for one matter only, or
(iii) Birbrower Exception: they are seeking to engage in ADR
that is related to something they are working on in their
authorized jurisdiction, or
(iv) Substantive Connection: the work they would be doing arises
out of, or is reasonably related to, their practice in the
authorized state.
(a) Comment 14: If a lawyer has recognized expertise in a
federal or otherwise nationally uniform area of the law, the
lawyer may practice in another jurisdiction (so long as he
does not establish a regular practice in that state).
(b) Host states have disciplinary jurisdiction over any lawyers
who perform or offer to perform legal services in the state.
(c) The ethics rules of the jurisdiction in which a tribunal sits
will apply to conduct in connection with matters pending
before that tribunal (choice-of-law provision)
(d) A lawyer will not be able to evade discipline by retreating
to his or her home state!
(b) Policies
(i) enables the regulatory structure for lawyers to catch up with the
reality of modern practice
(ii) ensures that states retain the ability to protect their residents
(iii) punishes lawyer misconduct within host states borders
3. Multidisciplinary Practice (MDP)
a. 5.4(a): Lawyers may not share legal fees with a nonlawyer
(i) Exception: Lawyers may hire other professionals to assist them (e.g.,
doctors, engineers, accountants, expert witnesses, analysts, lobbyists,
etc.) because it is considered paying salaries rather than splitting
profits
b. 5.4(d): A lawyer shall not practice in the form of an association if a
nonlawyer owns any interest therein.
(i) Example: a firm may not offer legal services, social work, and medical
services from a single office.
c. Policy: to protect the lawyers professional independence of judgment
(Comments 1 and 2 to Rule 5.4)
d. Not-for-Profit MDPs: These do not violate Rule 5.4!
4. Limited Representation

50
a. Rule 1.2: allows a lawyer to limit the scope of the representation if the
limitation is reasonable under the circumstances and the client gives
informed consent.
(i) Comment 6: Makes clear that a lawyer could legitimately offer limited
service to a client to exclude actions that the client thinks are too
costly.
(ii) Rule 6.5: allows a lawyer providing short-term services such as advice
or form completion under the auspices of a program sponsored by a
non-profit organization or court (e.g., call-in legal advice, Lawyer for
a Day) to do so without having to perform an extensive check for
conflicts of interest with other clients.
X. Legal Representation of an Entity
A. From Duggins Internal Corporate Investigations
1. Counsels first goal in conducting an internal investigation is to determine
whether wrongdoing has occurred. If it has, it is in the corporations best
interest to show that an errant employee or agent acted on her own without
either corporate encouragement or corporate authority.
a. Corporate counsel are ethically bound to choose the interests of the
corporation over those of its constituents. The employee, however, may
not realize his or her potential vulnerability.
B. The Privilege for Corporations (Organizational Privilege)
1. Upjohn Co. v. United States: held that, in federal proceedings applying federal
law, corporate entities could claim attorney-client privilege and that the scope
of the privilege should depend on the subject matter of the communication,
not on who was doing the communicating.
a. Control Group Test Rejected (but still used in some states): limits the
privilege to communications from persons in the organization who have
authority to mold organizational policy or to take action in accordance
with the lawyers advice (Restatement 73, comment d)
(i) Bad Policy: This test frustrates the very purpose of the privilege by
discouraging the communication of relevant information by employees
of the client to attorneys seeking to render legal advice to the client
corporation.
b. Subject Matter Test Endorsed: extends the privilege to communications
with any [management or] lower-echelon employee or agent so long as the
communication relates to the subject matter of the representation
(Restatement 73, comment d)
(i) Good Policy: This test encourages employees who have engaged in
wrongdoing to be candid.
(ii) Extension to Other Business Forms: The Restatement urges that
neither logic nor principle supports limiting the organizational
privilege to the corporate form (Restatement 73, comment b)
2. Samaritan Case
a. Narrows Upjohn: If a corporate counsel asks an employee about
something related to a case, and the employee speaks about something he

51
did, that is covered by the privilege; but if he speaks about something he
witnessed, that is not covered by the privilege.
C. Conflicts in Representation of Organization
1. Rule 1.13(b): Lawyers Duty to Act to Protect Organization
a. 3 Elements
(i) Lawyer must have knowledge
(ii) that someone in the company is either
(a) in violation of a legal obligation owed to the corporation, or
(i) Examples: shipping off trade secrets to a competitor;
embezzlement (looting, stealing company value)
(ii) These violations harm the company directly.
(b) in violation of a law that reasonably might be imputed to the
organization
(i) Example: polluting
(ii) These violations indirectly harm the company because,
although they may make the company financially better off,
they expose the company to risk of litigation.
(iii) in a way that is likely to result in substantial injury to the
organization.
b. Solution: Lawyer must proceed as is reasonably necessary in the best
interest of the corporation; must refer the matter to a higher authority in
the organization.
(i) Comment 5: Lawyer must notify the Board of Directors (or similar
governing body) of the violations, if others lower in the chain of
command fail to take appropriate action.
2. Rule 1.13(c): The Enron Addition
a. If the highest authority refuses to act properly, and the lawyer believes that
the misconduct will result in substantial injury to the organization, the
lawyer may reveal the misconduct to public officials.
(i) Sarbanes-Oxley Act
(a) Requires lawyers to disclose knowledge of material violations.
(i) Knowledge Standard is Objective: It must be reasonably likely
that the lawyer would become aware of evidence of the
material wrongdoing.
(ii) Broader Universe: than 1.13(b), which requires that the lawyer
believe the violation is reasonably certain to result in
substantial injury to the organization before he has a duty to
report
(a) But applies to fewer attorneys (only those who practice
before the SEC)
(b) Lawyer is required to report higher up (e.g., to the Board) more
quickly.
3. Client = Entity
a. NOT the officers
b. NOT the shareholders
c. NOT other corporations owned (in part or whole) by the corporation

52
4. Related Entities
a. More likely to be a client if
(i) the lawyer received confidential information from or provided advices
to the subsidiary
(ii) the entity was controlled and supervised by the parent organization
(iii) the original client could be harmed by the suit against the
subsidiary
b. Less likely to be a client if
(i) the lawyer no longer represents the initial corporate client
(ii) the two entities became linked (e.g., by a merger) after the lawyer
began representation of the corporation
c. Subsidiaries: Just because the parent company is your client, this does not
mean that its subsidiary is your client automatically (you must consider
factors) ==> you may be able to sue a clients subsidiary with no conflicts
issues. (Comment 34 to Rule 1.7)
(i) Factor: degree of independence between companies (e.g., overlap in
boards of directors?)
(a) Policy: you must assess the potential for access to confidential
information re: the subsidiary by virtue of representing the parent
(ii) No Direct Adversity: even if parent may be seriously harmed by a
verdict versus the subsidiary (according to the ABA)
5. Conflicts Guidelines
a. General: If you represent an entity, you need to obtain information about
other organizations and individuals that are related to the entity (owners
of, owned by, officers and directors of, and so forth) so that if you are
asked to take on a matter adverse to one of the related persons or entities,
you can evaluate whether there is a conflict.
b. When Sole Shareholder Controls: If a corporation-client has only one
shareholder, courts will consider representation of the corporation to be
joint with representation of that shareholder. See Tekni-Plex, Inc. v.
Meyner & Landis.
c. Employee Representation: A lawyer in private practice who represents an
organization may represent a member or an employee of the organization
unless the interests of the organization and the individual conflict
(i) Example: Corporate counsel may draft CEOs will.
(a) However: If counsel represents CEO prior to creation of
corporation (e.g., advising CEO how to set it up), then if counsel
stays on as corporate counsel upon corporations creation, he may
not represent third parties versus the CEO on unrelated matters
going forward because CEO would constitute a former client.
(ii) If consent for such a conflict is required, a designated corporate
official can give consent, so long as that person is not the one to be
represented by the lawyer.
d. Derivative Suits: In most cases, a lawyer for the organization should not
undertake to represent the defendants (officers and directors) in a
derivative suit, even with client consent.

53
(i) Exception: [I]f disinterested directors conclude that no basis exists for
the claims and all relevant persons consent, the lawyer may represent
the organization and the defendants.
(ii) NB: If the lawyer serves on the Board, and is accused of wrongdoing
in the suit, he may not represent the organization in the suit.
6. Duty to Protect Confidences of Employees
a. General Rule: If the lawyer represents the entity, he generally has no duty
to protect confidences of employees.
b. Solution: The lawyer must advise the employee that a potential conflict
exists (if one does) and that discussions between the employee and the
attorney are not confidential.
(i) Failure to give such a warning and lawyer-conduct which gives the
employee an impression that confidentiality exists might inadvertently
create a lawyer-client relationship with the employee ==> duty to
protect employees confidences AND a concurrent conflict of interest!
7. Entity Lawyers on Boards of Directors
a. General Rule: Lawyers are not forbidden to sit on boards of directors of
organizations that they represent; in many instances, there is no conflict
between the two roles.
(i) Exception: If a conflict arises that presents a substantial risk that the
lawyers representation of the client would be materially and adversely
affected by the lawyers obligations as a director, the lawyer should
cease to represent the corporation on that matter unless the
organization waives the conflict.
b. Attorney-Client Privilege: If a lawyer is also a director of an organization,
communications with him may not be protected by attorney-client
privilege.
XI. Legal Malpractice Doctrine
A. Four Elements of Legal Malpractice COA (from Togstad v. Vesely, Otto, Miller &
Keefe)
a. Existence of attorney-client relationship
b. Negligence or breach of contract by defendant
c. Proximate Cause: that the negligent acts or breach of contract was the
proximate cause of the plaintiffs damages
d. But For Cause: But for the defendants conduct, the plaintiffs would have
been successful in the prosecution of their legal claim or the negotiation of
their transactional deal (for which they sought the lawyers representation)
(i) Requires a showing that, but for the lawyers bad lawyering, the client
would have gotten a better deal/outcome.
(a) This is more difficult to show in transactional malpractice cases,
but it is not impossible.
B. Applies in litigation as well as transactional representations (see Viner)!
C. Limitations on Malpractice Liability
1. LLPs: permitted, provided that each lawyer remains personally liable to the
client for his or her own conduct and provided that the firm complies with
other restrictions imposed by law (Rule 1.18)

54
2. Contracts Prospectively Limiting Malpractice Liability: Rule 1.8(h)(1)
authorizes such limitations, but only if clients are represented by outside
counsel.
D. Settlement of Malpractice Claims: Rule 1.8(h)(2) permits a lawyer to settle a
malpractice claim with a client who does not have independent legal advice, but
the lawyer must advise the client in writing that it is a good idea to get advice
from another lawyer before making such a settlement.

55

Anda mungkin juga menyukai